terça-feira, 26 de outubro de 2010

O dolo e a culpa fazem parte da culpabilidade ou da tipicidade ou de ambas?

Inicialmente, cumpre registrar o que vem a ser dolo e culpa, culpabilidade e tipicidade. Dolo indica “sobretudo, vontade de produzir o resultado. Mas não é só. Também há dolo na conduta de quem, após prever e estar ciente de que pode provocar o resultado, assume o risco de produzi-lo” [1]. A culpa, por sua vez, constitui a inobservância de cuidado objetivo por parte do agente, seja por negligência, imprudência ou imperícia, isto é, atua através da criação de riscos proibidos [2].

A culpabilidade pressupõe “juízo de reprovação que recai sobre o agente do fato que podia se motivar de acordo com a norma e agir de modo diverso, conforme o Direito” [3], enquanto a tipicidade significa “a coincidência entre dado comportamento humano e a norma penal incriminadora” [4].

Feitas essas breves considerações, indaga-se: dolo e culpa fazem parte da culpabilidade, da tipicidade ou de ambas? Adentrando as teorias do Direito Penal, é possível fazer o seguinte esboço antes de se chegar a uma conclusão, uma opinião definitiva.

Pela teoria causal da ação (Von Liszt, Beling) adotava-se a teoria psicológica da culpabilidade. Esta, que tinha como pressuposto a imputabilidade, era “o vínculo existente entre o agente e seu delito, que se dava ou pelo dolo ou pela culpa. Dolo e culpa faziam parte da culpabilidade” [5]. Aqui, a culpabilidade é puramente psicológica, desprovida de “qualquer valoração e esgota-se na simples constatação da posição do agente perante sua própria conduta” [6].

A teoria neokantista [7], posteriormente, veio a conceber a “culpabilidade bipolarmente: ela é subjetiva e normativa ao mesmo tempo. Acolhe-se, então, a teoria psicológico-normativa da culpabilidade” [8]. Assim, a culpabilidade não era somente psicológica (imputabilidade + dolo ou culpa), mas também normativa (exigibilidade de conduta diversa), ou seja, “a culpabilidade agora é vista como juízo de reprovação, no entanto, não se transformara ainda num puro juízo de valoração” [9]; ainda assim, dolo e culpa continuavam a integrar a culpabilidade.

Já a teoria finalista da ação (Welzel) deslocou dolo e culpa para a tipicidade, afirmando que ambos “pertencem à conduta e em conseqüência ao fato típico: é requisito subjetivo ou normativo do tipo” [10]. Assim, a culpabilidade “transformou-se em juízo puramente normativo. É juízo que recai sobre o agente do fato” [11], daí que se adotou em nossa sistemática a teoria normativa pura da culpabilidade [12].

Uma quarta teoria, chamada teoria complexa da culpabilidade, sustenta estar dolo e culpa na tipicidade e na culpabilidade, ou seja, “o dolo e a culpa possuiriam dupla função dentro do Direito penal: fariam parte da tipicidade e também seriam valorados no âmbito da culpabilidade” [13]; daí ser chamada de complexa.

Pelo exposto, em que pese os entendimentos das teorias descritas, que muito contribuíram para a evolução do Direito Penal, o mais razoável é que a doutrina finalista supera as demais no que tange à culpabilidade, haja vista que o Direito Penal deve ser interpretado de modo a se valorar cada conduta, cada ação. Desse modo é que:

O Direito Penal não pode ser tido como uma mera ciência de proteção de interesses objetivos e formais. Muito pelo contrário, o Direito Penal não é apenas um catálogo de crimes e penas, mas sim e substancialmente o sustentáculo da ordem e garantia dos princípios fundamentais da vida organizada e para que o Direito Penal consiga cumprir sua relevante finalidade social, suas normas deverão ser sentidas, isto é, mais que compreendidas. [14]

Por derradeiro cumpre registrar que a culpabilidade, por ser juízo de desaprovação pela conduta do agente, não merece ter uma concepção meramente psicológica da culpabilidade, o que seria “uma concepção incolor, naturalística, fria, incapaz de adequar-se à rica casuística das situações para ver se é possível um juízo de reprovação e até que ponto”[15]. Portanto, como bem assinala nosso Código Penal, a teoria adotada acerca da culpabilidade é a do finalismo – embora não sejam observados de forma exclusiva os elementos dessa teoria em nosso Código-, eis que a culpabilidade é puro juízo de valor sobre o agente do fato. Isso explica o porquê de dolo e culpa serem afastados da culpabilidade e integrarem o tipo:

(...) sendo puro juízo de censura, não pode a culpabilidade abrigar em seu seio requisitos subjetivos ou psicológicos; logo, o dolo, que é psicológico, é afastado do âmbito da culpabilidade e passa a compor o tipo subjetivo dos delitos dolosos. A culpa, do mesmo modo, como forma de conduta humana, também passa a fazer parte do tipo nos crimes culposos. [16]

Referências

AMARAL JÚNIOR, Ronald. Culpabilidade como princípio. Disponível em: www.ibccrim.org.br.

GOMES, Luiz Flávio. Teorias causalista, finalista e constitucionalista do delito (síntese das distinções). Disponível em: www.lfg.com.br. Acesso em 9 julho 2010.

GOMES, Luiz Flávio, GARCÍA-PABLOS DE MOLINA, Antonio. Direito Penal: parte geral. 2. ed. São Paulo: RT, 2009, v. 2.

GOMES, Luiz Flávio, GARCÍA-PABLOS DE MOLINA, Antonio. BIANCHINI, Alice. Direito Penal: introdução e princípios fundamentais. São Paulo: RT, 2007, v. 1.

MASSON, Cleber Rogério. Direito penal esquematizado: parte geral. 2. ed. Rio de Janeiro: Forense; São Paulo: Método, 2009.

QUEIROZ, Paulo de Souza. Direito penal: parte geral. 6. ed. Rio de Janeiro: Lumen Juris, 2010.

NOTAS:

[1] MASSON, Cleber Rogério. Direito penal esquematizado: parte geral. 2. ed. Rio de Janeiro: Forense; São Paulo: Método, 2009, p. 248.

[2] Em linhas gerais, dolo e culpa têm seu significado extraído do artigo 18 do Código Penal.

[3] GOMES, Luiz Flávio, GARCÍA-PABLOS DE MOLINA, Antonio. Direito Penal: parte geral. 2. ed. São Paulo: RT, 2009, v. 2, pp. 408-414.

[4] QUEIROZ, Paulo de Souza. Direito penal: parte geral. 6. ed. Rio de Janeiro: Lumen Juris, 2010, p. 177.

[5] GOMES, Luiz Flávio, GARCÍA-PABLOS DE MOLINA, Antonio. Direito Penal: parte geral. 2. ed. São Paulo: RT, 2009, v. 2, pp. 408-414.

[6] Ibidem.

[7] Essa teoria significou “a última modificação sistemática do sistema naturalista, caracterizando-se pela visão normativa de valor do Direito Penal. É o abandono do naturalismo ou positivismo para a introdução no Direito Penal do normativismo axiológico”. GOMES, Luiz Flávio, GARCÍA-PABLOS DE MOLINA, Antonio. Op. cit.

[8] GOMES, Luiz Flávio, GARCÍA-PABLOS DE MOLINA, Antonio. BIANCHINI, Alice. Direito Penal: introdução e princípios fundamentais. São Paulo: RT, 2007, v. 1, pp. 520-539.

[9] GOMES, Luiz Flávio, GARCÍA-PABLOS DE MOLINA, Antonio. Direito Penal: parte geral. 2. ed. São Paulo: RT, 2009, v. 2, pp. 408-414.

[10] GOMES, Luiz Flávio. Teorias causalista, finalista e constitucionalista do delito (síntese das distinções). Disponível em: www.lfg.com.br. Acesso em 9 julho 2010.

[11] GOMES, Luiz Flávio, GARCÍA-PABLOS DE MOLINA, Antonio. BIANCHINI, Alice. Op. cit.

[12] “ O dolo e a culpa, assim, não fazem parte da culpabilidade; eles passam a ser “objeto da valoração” da culpabilidade. Eles integram o tipo e uma vez ausentes o fato é atípico”. GOMES, Luiz Flávio, GARCÍA-PABLOS DE MOLINA, Antonio. Op. cit.

[13] Ibidem.

[14] Apud AMERICANO, Odin. Da culpabilidade normativa. Estudos de Direito e Processo Penal em homenagem a Nélson Hungria. Rio de Janeiro-São Paulo: Forense, 1962, p. 359. AMARAL JÚNIOR, Ronald. Culpabilidade como princípio. Disponível em: www.ibccrim.org.br.

[15] Apud Bettiol. GOMES, Luiz Flávio, GARCÍA-PABLOS DE MOLINA, Antonio. Op. cit.

[16] Ibidem.


Felipe Pinto Bruno - Advogado inscrito na OAB/DF, Pós-graduando em Ciências Penais pelo curso LFG - Brasília.

segunda-feira, 20 de setembro de 2010

EMBARGOS DE DECLARAÇÃO NO PROCESSO DO TRABALHO

Aos amigos que acompanham e acessam o blog, gostaria de indicar um livro que acabou de ser lançado: EMBARGOS DE DECLARAÇÃO NO PROCESSO DO TRABALHO, de Vitor Salino de Moura Eça, lançado pela LTR.
Abaixo, segue breve resumo da obra:

"O processo não pode ser pensado sem o indispensável manejo dos embargos de declaração, que servem justamente para o aperfeiçoamento das decisões judiciais. O que pode ser mais importante?
Os embargos de declaração se constituem ainda em um dos recursos mais utilizados e, por incrível que possa parecer, um dos menos doutrinados.
Este livro vem suprir esta lacuna, oferecendo suporte tanto para o profissional, quanto para o estudioso.
Professores e juízes do trabalho reunidos sob a coordenação do professor doutor Vitor Salino de Moura Eça, vão muito além da simples normatividade. Discorrem sobre a origem do instituto, quais são as decisões embargáveis, o problema da irretratabilidade e da multa, assim como sobre o significado de vícios, obscuridades, contradições e omissões.
Questões processuais complexas, com os efeitos dos embargos, e o caráter infringente, bem como o prequestionamento são enfrentadas sem reservas, dando segurança na aplicação desse importante recurso.
Uma obra essencial."

Para maiores informações, segue o site:

http://www.ltreditora.com.br/lancamentos/embargos-de-declarac-o-no-processo-do-trabalho.html

Além de brilhante, e particularmente grande amigo, Vitor Salino de Moura Eça é doutor em Direito Processual e mestre em Direito do Trabalho, pela PUC-MG, além de Especialista em Direito Empresarial pela UGF e em Direito do Trabalho pela UBA. É juiz do trabalho no TRT da 3ª Região, Minas Gerais. Atuando como juiz convocado há muito tempo, participa com frequência de congressos e seminários, no Brasil e no exterior, assim como em bancas acadêmicas e nos concursos para juiz do trabalho substituto. Professor adjunto III da Pontifícia Universidade Católica de Minas Gerais, atua nos cursos de graduação, especialização, mestrado e doutorado em Direito. É conselheiro da Escola Judicial e professor visitante em diversas universidades, tais como: Universidad Nacional de Córdoba, Faculdade de Direito de Vitória/ES, Faculdade Pitágoras, dentre outras. Membro efetivo das seguintes sociedades: Academia Brasileira de Direito Constitucional – ABDConst – Paraná/PR; Asociación Iberoamericana de Derecho del Trabajo y de la Seguridad Social – AIDTSS; Associação dos Magistrados Brasileiros – AMB – Brasília/DF; Associação dos Magistrados Mineiros – AMAGIS – Minas Gerais/MG; Associação dos Magistrados Trabalhistas da 3ª Região – AMATRA III – Minas Gerais/MG; Conselho Nacional de Pesquisa e Pós-Graduação em Direito – CONPEDI – Florianópolis/SC; Academia Brasileira de Direito Processual Civil – ABDPC – Porto Alegre/RS; Associação Nacional dos Magistrados da Justiça do Trabalho – ANAMATRA – Brasília/DF; Associação Latino-Americana de Juízes do Trabalho – ALJT – Brasília/DF; Equipo Federal del Trabajo – EFT – Buenos Aires/Argentina; Fórum Mundial de Juízes – FMJ – Brasília/DF; Instituto Brasileiro de Direito Social Júnior – IBDSCJ – São Paulo; Societé Internationale de Droit du Travail et de la Sécurité Sociale; Rede Brasileira de Juízes – RBJ – Brasília/DF e da Red Latinoamericana de Jueces para Cooperación Judicial e Integración – España/Brasil.

Fica aqui a indicação!

sábado, 28 de agosto de 2010

O porte de arma desmuniciada constitui crime?

O tipo penal do artigo 14 da lei 10.826/03 prescreve que "Portar, deter, adquirir, fornecer, receber, ter em depósito, transportar, ceder, ainda que gratuitamente, emprestar, remeter, empregar, manter sob guarda ou ocultar arma de fogo, acessório ou munição, de uso permitido, sem autorização e em desacordo com determinação legal ou regulamentar" configura crime, um ilícito penal de mera conduta e de perigo abstrato, ou seja, aquele em que não é necessário qualquer resultado material, bem como o perigo é presumido, eis que o simples fato de portar a arma (para não citar todos os verbos do núcleo do tipo, posto que aqui se trata especificamente do porte), mesmo desmuniciada, é suficiente para condenação criminal.

Para adentrar melhor na questão (polêmica), destaca-se o seguinte trecho do voto proferido pela Ministra Ellen Gracie no HC 99.449/MG, in verbis:

"O fato de estar desmuniciada não a desqualifica como arma, tendo em vista que a ofensividade de uma arma de fogo não está apenas na sua capacidade de disparar projéteis, causando ferimentos graves ou morte, mas também, na grande maioria dos casos, no seu potencial de intimidação" [1].

No citado Habeas Corpus, a relatora decidiu pela tipicidade da conduta, denegando, consequentemente, a ação impetrada, sob a justificativa de que o legislador, nos crimes de perigo abstrato, busca "antecipar a punição de fatos que apresentam potencial lesivo à população - como o porte de arma de fogo em desacordo com as balizas legais -, prevenindo a prática de crimes como homicídios, lesões corporais, roubos etc.", portanto "sendo irrelevante o fato de a arma estar carregada ou não" [2].

Aludido entendimento, data venia, configura uma posição legalista [3], verdadeira interpretação da letra fria da lei. Em que pese o voto da eminente relatora, esse entendimento não deve prosperar, haja vista ser inconstitucional "pois não se pode restringir direitos fundamentais básicos como a liberdade ou o patrimônio sem que seja para tutelar concretas ofensas a outros direitos fundamentais" [4].

O porte de arma desmuniciada deve ser analisado sob o prisma da ofensividade (também conhecido como princípio da lesividade), que traduz o seguinte:

"O fato cometido, para se transformar em fato punível, deve afetar concretamente o bem jurídico protegido pela norma; não há crime sem lesão ou perigo concreto de lesão ao bem jurídico tutelado - nullum crimen sine injuria" [5].

A norma jurídica penal deve ser lastreada por um aspecto valorativo, ou seja, a norma existe para a proteção de um valor que fundamenta o injusto penal; coligado com o princípio da ofensividade, afirma-se que o crime exige desvalor da ação, caracterizado pela realização de uma conduta valorada negativamente, bem como o desvalor do resultado, caracterizado pela afetação concreta de um bem jurídico [6].

Isso demonstra que a norma penal deve ser valorada ao ser interpretada, caso a caso (exigência da tipicidade objetiva material). Ademais, atualmente a tipicidade é analisada com base em diversas exigências [7], sendo que na ausência de qualquer delas o fato é atípico.

Nesse contexto, não há que se falar em crime quando não se evidencia materialidade delitiva, porquanto não há ofensividade; esse princípio confirma a ausência de perigo abstrato no âmbito do Direito Penal. Assim é que para os que não consideram o princípio da ofensividade, o porte de arma desmuniciada configura crime [8].

Pela posição constitucionalista (teoria esta que é muito mais garantista [9]) para ser considerado um crime, este deve se revestir de ofensividade (lesividade) ao bem jurídico protegido, pois a "norma existe para tutelar um bem jurídico e sem ofensa a esse bem não há delito" [10].

Vivemos num Estado Democrático de Direito em que o Direito Penal deve ser lido através da Constituição Federal, o que significa dizer que os direitos fundamentais não podem sofrer violação (em regra), de modo que outros ramos do Direito podem e devem cuidar de casos de atos lesivos com a mesma eficiência [11], deixando para a órbita penal somente quando houver efetiva e concreta lesão a bens jurídicos mais importantes, pois o Direito Penal configura o remédio mais extremo de punição dentro de um ordenamento jurídico, por implicar na privação da liberdade dos indivíduos [12].

Assim, como não há crime sem ofensa ao bem jurídico, o porte de arma desmuniciada não deve configurar um delito, haja vista que sequer se constata nesta conduta um resultado jurídico lesivo, sendo inadmissível um resultado jurídico presumido.

Referências:

CANAL, Verônica Correia. Atipicidade dos crimes de porte de arma desmuniciada e a posse de munição: exclusão da tipicidade material. Disponível em: http://www.ibccrim.org.br/.

GOMES, Luiz Flávio. Princípios constitucionais reitores do direito penal e da política criminal. Disponível em: http://www.lfg.com.br/

SANTOS, Carla Maia. O porte de arma desmuniciada e a posse de munição. Disponível em: http://www.lfg.com.br/

Notas:

[1] Nesse caso, entendo que num eventual crime de roubo, por exemplo, a utilização de arma sem munição pelo agente se enquadraria no caput do artigo 157, em forma de grave ameaça, e não no parágrafo segundo do mesmo dispositivo, pois sequer possui potencialidade lesiva.

[2] No mesmo sentido vide HC 147.623/RJ e HC 90.197/DF.

[3] "Os legalistas admitem o perigo abstrato. Os constitucionalistas refutam esse modelo de perigo. Para os legalistas o porte de arma sem munição é delito. Para os constitucionalistas o relevante é o perigo concreto". SANTOS, Carla Maia. O porte de arma desmuniciada e a posse de munição. Disponível em: http://www.lfg.com.br/. Acesso em: 10 jun. 2010.

[4] GOMES, Luiz Flávio. Princípios constitucionais reitores do direito penal e da política criminal. Disponível em: http://www.lfg.com.br/. Acesso em: 10 jun 2010.

[5] Ibidem.

[6] Ibidem.

[7] Tipicidade objetiva formal, tipicidade objetiva material e tipicidade subjetiva.

[8] GOMES, Luiz Flávio. Princípios constitucionais reitores do direito penal e da política criminal. Disponível em: http://www.lfg.com.br/. Acesso em: 10 jun. 2010.

[9] O Sistema Garantista, resumidamente, caracteriza-se pela tutela dos direitos fundamentais e limitação do poder estatal.

[10] GOMES, Luiz Flávio. Princípios constitucionais reitores do direito penal e da política criminal. Disponível em: http://www.lfg.com.br/. Acesso em: 10 jun. 2010.

[11] Trata-se do princípio da necessidade, integrante do Sistema Garantista, consistente no fato de que o Direito Penal não deve intervir sempre, ele é ultima ratio.

[12] CANAL, Verônica Correia. Atipiciade dos crimes de porte de arma desmuniciada e a posse de munição: exclusão da tipicidade material. Disponível em: http://www.ibccrim.org.br/. Acesso em: 10 jun. 2010.

Felipe Pinto Bruno - Advogado inscrito na OAB/DF, Pós-graduando em Ciências Penais pelo curso LFG - Brasília.

quinta-feira, 24 de junho de 2010

EXAME DA OAB: FLAGRANTE INCONSTITUCIONALIDADE E A REPERCUSSÃO GERAL

Muito se tem discutido, desde a sua implantação em 1996, a respeito do “exame” da OAB. Basicamente temos duas vertentes de sustentação, a parte dos Bacharéis em Direito, contra a aplicação do exame, num flagrante cerceamento ao constitucional exercício profissional, e a OAB, defendendo a aplicação do exame como forma de se saber se o bacharel está apto para o exercício da advocacia.
O que se tem observado no decorrer dos últimos anos é que além dos bacharéis em Direito, muitos outros segmentos da sociedade vem numa luta ferrenha contra a aplicação do tal exame. Há inúmeros projetos que visam a retirada da obrigatoriedade do exame para o exercício, repito, constitucional da atividade profissional.
Também, no próprio ceio dos advogados, existem aqueles que pugnam pela retirada do exame. Vemos ainda na classe dos professores, dos juízes e parlamentares, posicionamentos favoráveis a retirada do exame de ordem. Há vários projetos de lei com esta finalidade tramitando no Congresso Nacional, entre eles podemos citar: PL 2.195/07; PL 2.246/07 e o PL do Senado 186/06, que revoga o inc IV e o § 1º do art. 8º da Lei Federal nº 8.906/04. Em todos os projetos apresentados a justificativa é a inconstitucionalidade da obrigatoriedade do exame de ordem, violação ao art. 5º, inc VIII, art. 205 e art. 22, inc XVI todos da Magna Carta.

Ainda no ano passado, o Senado Federal se manifestou no sentido de promover audiência pública sobre o exame de ordem, para a discussão do PL 186/06 de autoria do senador Gilvam Borges, PMDB/AP, muito embora ainda não se tenha uma data prevista.

Inúmeras ações judiciais vem contestando a aplicação do exame de ordem e recentemente, uma decisão na Justiça Federal do Rio de Janeiro, deu liminar para que seis bacharéis em Direito[1] se inscrevessem na OAB, mesmo sem aprovação no exame:
A Justiça Federal do Rio de Janeiro permitiu que seis bacharéis em Direito atuem como advogados mesmo sem aprovação no Exame de Ordem. Em janeiro de 2008, a juíza Maria Amélia Almeida Senos de Carvalho, da 23ª Vara Federal, concedeu liminar para permitir a inscrição na OAB. Ao analisar o mérito da questão, em fevereiro deste ano, concluiu que exigir que o bacharel seja submetido ao exame para poder trabalhar é inconstitucional.
Posteriormente, o presidente do Tribunal Regional Federal da 2ª Região, suspendeu a decisão da Juíza Federal Maria Amélia Almeida Senos de Carvalho, em recurso interposto pela OAB, o Des. Castro Aguiar declarou que não entraria na discussão se a decisão da juíza Maria Amélia estaria certa ou errada, apenas que a sua decisão era para preservar o interesse público, ainda explicando que apenas suspendeu os efeitos da decisão da juíza, não a reforma e nem a cassa. Desse fato, observamos, mais uma vez a vitória do corporativismo e da política desempenhada pela OAB.

Recentemente o Min. Marco Aurélio, relator do RE 603.583-RS, recurso este, que contesta a decisão do TRF-4, na qual afirma que somente bacharéis em Direito podem participar do exame de ordem, manifestou-se no sentido da Repercussão Geral sobre o tema, e os ministros do Supremo por unanimidade acolheram o voto do relator, reconhecendo que há Repercussão Geral no Recurso Extraordinário. Decisão essa que passa por contornos de controle de constitucionalidade.

Em breve o STF vai se manifestar sobre a questão da inconstitucionalidade do exame da OAB, que pretere bacharéis de Direito, já diplomados pelo MEC, ao exercício profissional da advocacia, presente no texto constitucional na forma do art. 5º, inc XIII de nossa Magna Carta.

O Recurso Extraordinário está previsto no art. 102[2], inc III da Magna Carta e permite a impugnação pelo STF das decisões em única ou última instância, que envolvam matéria constitucional:
Art. 102. Compete ao Supremo Tribunal Federal, precipuamente, a guarda da Constituição, cabendo-lhe:
III - julgar, mediante recurso extraordinário, as causas decididas em única ou última instância, quando a decisão recorrida:
a) contrariar dispositivo desta Constituição;
b) declarar a inconstitucionalidade de tratado ou lei federal;
c) julgar válida lei ou ato de governo local contestado em face desta Constituição.
d) julgar válida lei local contestada em face de lei federal.
O Recurso Extraordinário é a penúltima etapa do controle difuso de constitucionalidade. A questão é suscitada incidenter tantum, ou seja, de modo incidental, o objeto da ação não é a constitucionalidade em si, mas uma relação jurídica envolvendo lei cuja validade frente à Constituição possa ser argüida, onde a solução da questão constitucional é fundamental para a decisão do litígio.
A questão é submetida ao Plenário do Supremo, com quorum de oito ministros e a decisão deve ser da maioria com o voto de no mínimo seis dos ministros. A decisão gera efeitos inter partes e ex tunc. Somente o Senado, após o trânsito em julgado da decisão favorável do STF, pode conferir efeito erga omnes a decisão, como prevê o art. 52, inc X da Constituição Federal, suspendendo a lei ou artigos da lei em questão.
Hodiernamente há uma nova tendência doutrinária e jurisprudencial de equiparação dos efeitos da decisão do controle difuso às do controle concentrado, ou seja, o efeito da decisão que declara a inconstitucionalidade de determinada norma pelo Supremo no exame de um Recurso Extraordinário não ficaria restrito somente ao caso em concreto analisado, reforçando a idéia de que o Supremo Tribunal Federal não pode acumular funções de um Tribunal Constitucional e ao mesmo tempo julgar causas cujos efeitos de sua decisão somente repercutirão entre as partes envolvidas, uma vez que a função precípua desse Tribunal é justamente a de guardião da Constituição.

O Min. Gilmar Mendes[3] endossa a abstrativização do controle difuso e declara que:
(...) marca uma evolução no sistema de controle de constitucionalidade brasileiro, que passa a equiparar, ainda que de forma tímida, os efeitos das decisões proferidas nos processos de controle abstrato e concreto.
Noutra passagem o Min. Gilmar Mendes[4] se manifestou nesse sentido no Processo Administrativo n. 318.715/STF:
O recurso extraordinário ‘deixa de ter caráter marcadamente subjetivo ou defesa de interesse das partes, para assumir, de forma decisiva, a função de defesa da ordem constitucional objetiva. Trata-se de orientação que os modernos sistemas de Corte Constitucional vêm conferindo ao recurso de amparo e ao recurso constitucional (Verfassungsbeschwerde). (...)A função do Supremo nos recursos extraordinários – ao menos de modo imediato - não é a de resolver litígios de fulano ou beltrano, nem a de revisar todos os pronunciamentos das Cortes inferiores. O processo entre as partes, trazido à Corte via recurso extraordinário, deve ser visto apenas como pressuposto para uma atividade jurisdicional que transcende os interesses subjetivos.
O que se depreende é que vem ocorrendo verdadeira mutação constitucional, ou seja, vem ocorrendo alterações no significado e sentido interpretativo de um texto constitucional. De forma que nessa nova interpretação o Senado somente possuiria o ônus da publicidade, ou seja, apenas o dever de divulgar a suspensão da execução, no todo ou em parte, de lei declarada inconstitucional por decisão definitiva, transitada em julgado, pelo STF.
Sem dúvida esse novo posicionamento do STF, dentro do controle de constitucionalidade, reflete a sua missão de guardião da Constituição.
Convém destacar que, pós Emenda Constitucional 45, houve mudanças no sentido de uma seleção dos recursos a serem apreciados pelo STF, com a criação do instituto da Repercussão Geral.
O instituto da Repercussão Geral é um instrumento processual que possibilita ao STF a seleção dos Recursos Extraordinários que irá analisar, de acordo com os critérios de relevância jurídica, política, social ou econômica. Trata-se na verdade de um filtro recursal que, por certo, tem a finalidade de diminuir a quantidade de processos encaminhados à Suprema Corte. No site do Supremo[5] temos a seguinte definição acerca da Repercussão Geral:
Repercussão Geral
Descrição do Verbete: A Repercussão Geral é um instrumento processual inserido na Constituição Federal de 1988, por meio da Emenda Constitucional 45, conhecida como a “Reforma do Judiciário”. O objetivo desta ferramenta é possibilitar que o Supremo Tribunal Federal selecione os Recursos Extraordinários que irá analisar, de acordo com critérios de relevância jurídica, política, social ou econômica. O uso desse filtro recursal resulta numa diminuição do número de processos encaminhados à Suprema Corte. Uma vez constatada a existência de repercussão geral, o STF analisa o mérito da questão e a decisão proveniente dessa análise será aplicada posteriormente pelas instâncias inferiores, em casos idênticos. A preliminar de Repercussão Geral é analisada pelo Plenário do STF, através de um sistema informatizado, com votação eletrônica, ou seja, sem necessidade de reunião física dos membros do Tribunal. Para recusar a análise de um RE são necessários pelo menos 8 votos, caso contrário, o tema deverá ser julgado pela Corte. Após o relator do recurso lançar no sistema sua manifestação sobre a relevância do tema, os demais ministros têm 20 dias para votar. As abstenções nessa votação são consideradas como favoráveis à ocorrência de repercussão geral na matéria.
Por verdade, há uma inegável tendência de que as instâncias inferiores sigam a decisão do STF, muito embora não exista vinculação a esse respeito no caso do Recurso Extraordinário.

Sobre esse instituto o que mais nos interessa neste momento é a declaração por unanimidade do Plenário do STF, da existência de Repercussão Geral da questão constitucional suscitada, em 11.12.2009, no RE 603.583-RS[6], da lavra do Relator Min. Marco Aurélio que assim se pronunciou:
"RELATOR : MIN. MARCO AURÉLIO
RECTE.(S): JOÃO ANTÔNIO VOLANTE
ADV.(A/S): CARLA SILVANA RIBEIRO D AVILA
RECDO.(A/S): UNIÃO
ADV.(A/S): ADVOGADO-GERAL DA UNIÃO
RECDO.(A/S): CONSELHO FEDERAL DA ORDEM DOS ADVOGADOS DO BRASIL
ADV.(A/S): MIRIAM CRISTINA KRAICZK E OUTRO(A/S)
PRONUNCIAMENTO
EXAME DE ORDEM – LEI Nº 8.906/94 – CONSTITUCIONALIDADE ASSENTADA NA ORIGEM – RECURSO EXTRAORDINÁRIO E RECURSO ESPECIAL – TEMA ÚNICO DE ÍNDOLE CONSTITUCIONAL – REPERCUSSÃO GERAL CONFIGURADA.
1. A Assessoria prestou as seguintes informações:
Submeto a Vossa Excelência o tema veiculado no Recurso Extraordinário nº 603.583/RS, para exame da oportunidade de incluir a matéria no sistema eletrônico da repercussão geral.
O Tribunal Regional Federal da 4ª Região rejeitou a alegação de inconstitucionalidade do artigo 8º, § 1º, da Lei nº 8.906/94 e dos Provimentos nº 81/96 e 109/05 do Conselho Federal da Ordem dos Advogados do Brasil. Conforme a Corte, ao estabelecer que somente bacharéis em Direito podem participar do Exame de Ordem, o Conselho Federal da OAB observou os limites da competência prevista no mencionado preceito legal. Além disso, a exigência de aprovação no Exame de Ordem como requisito para o exercício da advocacia não conflitaria com o princípio da liberdade profissional – artigo 5º, inciso XIII, da Carta da República. Os embargos declaratórios interpostos contra o acórdão foram desprovidos.
No extraordinário interposto com alegada base na alínea a do permissivo constitucional, o recorrente articula com a ofensa aos artigos 1º, incisos II, III e IV, 3º, incisos I, II, III e IV, 5º, incisos II e XIII, 84, inciso IV, 170, 193, 205, 207, 209, inciso II, e 214, incisos IV e V, da Lei Maior. Inicialmente, afirma não haver pronunciamento do Supremo quanto à constitucionalidade do Exame de Ordem. Sustenta caber apenas à instituição de ensino superior certificar se o bacharel é apto para exercer as profissões da área jurídica. Reputa inconstitucional a autorização, constante do artigo 8º da Lei nº 8.906/94, para regulamentação do Exame de Ordem pelo Conselho Federal da Ordem dos Advogados do Brasil, consideradas a afronta ao princípio da legalidade e a usurpação da competência privativa do Presidente da República para regulamentar leis.
Conforme alega, a submissão dos bacharéis ao Exame de Ordem atenta contra os princípios constitucionais da dignidade da pessoa humana, da igualdade bem como do livre exercício das profissões e contra o direito à vida. Impedir que os bacharéis exerçam a profissão de advogado após a conclusão do curso universitário também representaria ofensa aos princípios da presunção de inocência, do devido processo legal, do contraditório e da ampla defesa. Discorre sobre o valor social do trabalho, fundamento da República Federativa do Brasil, e diz que a exigência de aprovação no Exame de Ordem representa censura prévia ao exercício profissional. Por fim, relata ter sido editada norma federal específica com a finalidade de regulamentar, para todas as profissões, o artigo 205 da Carta da República: a Lei nº 9.394/96, denominada Lei de Diretrizes e Bases da Educação Nacional.
Sob o ângulo da repercussão geral, assevera que o entendimento a ser adotado por esta Corte norteará a aplicação do Direito Constitucional em inúmeros casos semelhantes. Afirma estar em jogo tema relevante do ponto de vista: a) moral, diante da frustração dos bacharéis impedidos de exercer a advocacia e dos respectivos familiares; b) econômico, pois a carteira de advogado viabilizaria o exercício da profissão e c) social, considerada a impossibilidade de o bacharel participar efetivamente da sociedade como conhecedor e aplicador do Direito. Diz da existência de vários projetos de lei a respeito da extinção do Exame de Ordem.
O Vice-Presidente da Corte de origem admitiu o extraordinário.
O trânsito do recurso especial simultaneamente protocolado foi obstado na origem. Não há notícia da interposição de agravo de instrumento dirigido ao Superior Tribunal de Justiça.
Brasília, 5 de novembro de 2009.
2. A ausência de interposição de agravo contra o ato que implicou a inadmissibilidade do recurso especial não prejudica o trânsito deste extraordinário. A razão é única: o acórdão impugnado tem fundamento estritamente constitucional, havendo a Corte de origem placitado a Lei nº 8.906/94.
No mais, está-se diante de situação concreta retratada em inúmeros processos. Bacharéis em Direito insurgem-se nos diversos órgãos do Judiciário contra o denominado Exame de Ordem, que, segundo argumentam, obstaculiza de forma setorizada, exclusivamente quanto a eles, o exercício profissional. O Supremo há de pacificar a matéria, pouco importando em que sentido o faça.
3. Manifesto-me pela existência de repercussão geral.
4. Incluam no sistema.
5. À Assessoria, para acompanhar o incidente.
6. Publiquem.
Brasília – residência –, 14 de novembro de 2009, às 20h."
E assim terminou o pronunciamento do Min. Marco Aurélio, óbvio está que o STF vai pacificar a matéria, resguardando a sua primordial função de guardião da Magna Carta. E por certo esperamos que o Supremo julgue no sentido da inconstitucionalidade, extirpando do nosso ordenamento jurídico essa afronta a princípios constitucionais tão evidentes. Assim também se manifesta, Fernando Machado de Silva Lima[7], advogado e professor de Direito Constitucional da UNAMA e principal articulista pela inconstitucionalidade do exame de ordem:
Evidentemente, o Supremo Tribunal Federal deverá pacificar a matéria, no desempenho de sua função de guardião da Constituição. Não é mais possível que o Exame da OAB continue sendo aplicado aos bacharéis em direito, apenas aos bacharéis em direito, sob a injurídica, capciosa, desonesta e ridícula alegação de que ele é necessário, para resguardar os interesses da sociedade contra advogados que não tivessem a mínima qualificação profissional.
Não é possível que qualquer pessoa medianamente inteligente não consiga entender que esse exame fere o princípio constitucional da igualdade. Apenas para exemplificar: se um exame desse tipo fosse necessário na área jurídica, com maior razão ainda ele seria necessário para os médicos e para os engenheiros. Isso é evidente, e dispensa maiores explicações.
Vamos com certeza aguardar essa decisão do Supremo Tribunal Federal, tão preciosa para os bacharéis de Direito.
A Lei nº 8.906/94[8] prevê em seu art. 8º, inc IV, para a inscrição do bacharel de Direito nos quadros da OAB, a aprovação em exame de ordem, e em seu §1º prevê a regulamentação do exame pelo Conselho Federal da OAB, conforme se transcreve:
Art. 8º Para inscrição como advogado é necessário:
IV - aprovação em Exame de Ordem;
§ 1º O Exame da Ordem é regulamentado em provimento do Conselho Federal da OAB.
A nosso sentir tais previsões guardam em si flagrantes inconstitucionalidades, tanto no aspecto formal, quanto no aspecto material. Em primeiro lugar a Magna Carta é a lei suprema do nosso Estado Democrático de Direito, e somente ela pode delegar poderes e competências. O exame de ordem é regulado por provimento do Conselho Federal da OAB, e temos claramente definido no art. 84, inc IV da nossa Constituição Federal que é competente, privativamente, o Presidente da República, para a regulação de leis, conforme se transcreve:
Art. 84. Compete privativamente ao Presidente da República:
IV - sancionar, promulgar e fazer publicar as leis, bem como expedir decretos e regulamentos para sua fiel execução;
Em assim sendo, atividade privativa do Presidente da República, a Lei n. 8.906/94 não pode atribuir competência ao Conselho Federal da OAB para a regulamentação do exame de ordem, portanto é inconstitucional a delegação, e em decorrência deste aspecto o provimento n. 109/05, ato administrativo, expedido pelo referido Conselho é também, por via reflexa, inconstitucional. Logo, temos aí configurada uma inconstitucionalidade formal do exame de ordem, uma vez que não compete ao Conselho Federal da OAB o poder de regulamentar leis federais.
Mas cabe ainda a análise da inconstitucionalidade material, uma vez que o exame de ordem vai de encontro ao art. 5º, inc XIII da Constituição, com previsão de que é livre o exercício de qualquer trabalho, ofício ou profissão, atendidas as qualificações profissionais que a lei estabelecer. Nessa cláusula pétrea infere-se que o bacharel de Direito, qualificado por Instituição de Ensino Superior, não poderia de forma alguma se submeter a exame de ordem, como condição do seu livre exercício profissional, vez que na expressão “qualificação profissional que a lei estabelecer” não se enquadra a aplicação do referido exame. A qualificação profissional é definida pelas Instituições de Ensino Superior, reconhecidas e fiscalizadas pelo Poder Público, diga-se MEC.
Ainda nessa seara, apontamos a afronta ao principio constitucional da dignidade da pessoa humana, uma vez que o exame de ordem impede o exercício da advocacia e o direito ao trabalho, aos bacharéis de Direito plenamente qualificados ao exercício profissional, através das Instituições de Ensino que lhes conferiram a diplomação, atendidas as exigências do MEC, numa violação flagrante ao art. 1º, incs III e IV da Constituição Federal.
Por derradeiro, apontamos ainda ofensa ao principio constitucional da igualdade, haja vista que qualquer bacharel, em nosso país, pode exercer livremente a sua profissão, necessitando apenas solicitar a sua inscrição no respectivo conselho profissional, como é o caso dos professores, dos médicos, dos dentistas, e tantas outras profissões. Mas isto não se aplica aos bacharéis de Direito, estes são os únicos no país obrigados a efetuar um exame de ordem, mesmo tendo sido submetidos a estágio profissional, também regulado pela OAB, e concluído a graduação, após cinco longos anos de estudos.

Em nosso país a gestão do ensino deve atender aos preceitos contidos na Lei n. 9.394/96[9]. A Lei de Diretrizes e Bases fixou como finalidade da educação superior a formação de diplomados nas diferentes áreas do conhecimento, aptos dessa forma a inserção nos setores profissionais, assim prevê seu art. 43, inc II, que se transcreve:
Art. 43. A educação superior tem por finalidade:
II - formar diplomados nas diferentes áreas de conhecimento, aptos para a inserção em setores profissionais e para a participação no desenvolvimento da sociedade brasileira, e colaborar na sua formação contínua;
Ora, vê-se clara e cristalinamente, que é a Instituição de Ensino Superior a competente para a formação de profissionais nas diversas áreas do conhecimento. Tal atribuição legal é frontalmente violada pelo exame de ordem, uma vez que este, tem a pretensão de qualificar os bacharéis de direito ao exercício profissional da advocacia, cuja clara competência é das Instituições de Ensino Superior, do Estado, via MEC.
Trazemos a colação o art. 48 da Lei de Diretrizes e Bases, que determina que os diplomas de cursos superiores reconhecidos, quando registrados, tem validade nacional como prova de formação profissional, sendo expedidos e registrados pelas próprias universidades, assim é a previsão:
Art. 48. Os diplomas de cursos superiores reconhecidos, quando registrados, terão validade nacional como prova da formação recebida por seu titular.
§ 1º Os diplomas expedidos pelas universidades serão por elas próprias registrados, e aqueles conferidos por instituições não-universitárias serão registrados em universidades indicadas pelo Conselho Nacional de Educação.
Ainda o art. 53, incs I, II e VI do mesmo diploma dispõe a competência das Universidades para fixar currículos de seus cursos e programas, e conferir graus, diplomas e outros títulos, numa clara referência a autonomia das Instituições de Ensino Superior:
Art. 53. No exercício de sua autonomia, são asseguradas às universidades, sem prejuízo de outras, as seguintes atribuições:
I - criar, organizar e extinguir, em sua sede, cursos e programas de educação superior previstos nesta Lei, obedecendo às normas gerais da União e, quando for o caso, do respectivo sistema de ensino; (Regulamento)
II - fixar os currículos dos seus cursos e programas, observadas as diretrizes gerais pertinentes;VI - conferir graus, diplomas e outros títulos;
Não resta a menor dúvida da invasão de competência firmada pelo exame de ordem. Os bacharéis de Direito tem sua qualificação profissional definida pela Lei de Diretrizes e Bases em comento, via Instituição de Ensino Superior e não devem se submeter ao exame de ordem, como condição de exercício profissional, uma vez que a OAB não tem competência para avaliar a qualificação profissional do bacharel de Direito.

Nesse sentido trazemos a colação o art. 205, que encerra claramente que a educação é dever do Estado e que qualifica o bacharel para o trabalho, e o art. 209[10], onde fica claro que a autorização e a avaliação da qualificação profissional compete ao Poder Público, via Ministério da Educação e Cultura, ambos da Magna Carta:
Art. 205. A educação, direito de todos e dever do Estado e da família, será promovida e incentivada com a colaboração da sociedade, visando ao pleno desenvolvimento da pessoa, seu preparo para o exercício da cidadania e sua qualificação para o trabalho.
Art. 209. O ensino é livre à iniciativa privada, atendidas as seguintes condições:
I - cumprimento das normas gerais da educação nacional;II - autorização e avaliação de qualidade pelo Poder Público.
Mais uma vez é firmada a inconstitucionalidade do exame de ordem da OAB.

É claro que a OAB defende o exame de ordem com unhas e dentes, é uma mina de ouro para a entidade. Vejamos que anualmente temos a aplicação de três exames de ordem, com a taxa de inscrição a R$ 145,00, valor do 1º exame de 2009, e tendo um índice médio de aprovação inferior a 20%, certamente somos forçados a concluir que mais de 80% dos inscritos farão novamente o próximo exame. É uma bola de neve. Isso sem mencionar os cursinhos jurídicos.

A OAB, até o presente momento, defende a aplicação do exame de ordem e para tal, argumenta que há uma gigantesca proliferação de cursos jurídicos, que o ensino jurídico nas Universidades é muito deficiente e que o MEC não fiscaliza os cursos superiores. Em palestra realizada no tema da XXXIX Colégio de Presidentes das Subseções da OAB-MS[11], o presidente da OAB-RJ, Sr. Wadhi Nemer Damous Filho declarou que:
"Atualmente existem 1.085 cursos jurídicos no Brasil. Só em meu estado (RJ) são 110 cursos. Só para comparar, em todos os EUA existem 220 cursos. Esse excesso de instituições de ensino jurídico brasileiro infelizmente não é acompanhado de qualidade acadêmica. Na maioria das vezes, como ocorreu no Rio, são cursos montados em qualquer 'biboca' de esquina por empresários que mercantilizam o ensino", afirmou Damous. "Temos de destacar que sob a gestão do presidente Cezar Britto o Conselho Federal da OAB não tem medido esforços em gestões junto ao Ministério da Educação contra a abertura de novos cursos e pelo rigor quanto à qualidade dos cursos já existentes, mas, lamentavelmente, não há um retorno adequado à estas gestões por parte do MEC."
Nesse ponto precisamos lembrar que o exame de ordem não é uma construção pátria, foi inspirado no modelo americano, ou seja, mais uma vez se incorporam as construções alienígenas. A guisa de informação temos que o exame de ordem também é aplicado em muitos países, como na Itália, Alemanha, Portugal, Japão, Suíça, Áustria, Inglaterra, França e tantos outros. Mas é por demais importante dizer que a aplicação desse exame é privativa do Poder Público e não do órgão de classe, isso a OAB não menciona em suas alegações.

O que na verdade estamos presenciando é uma manobra astuta para a obtenção e manutenção de arrecadação. Segundo o relatório de desempenho das Instituições de Ensino Superior, elaborado pelo CESPE e apresentado no próprio site da OAB[12], referente ao exame de ordem 2009/1 nacional, o número total de inscritos foi de 59.832, desses 58.761 prestaram a primeira fase do exame, com 12.857 aprovados para a segunda fase e sendo aprovados apenas 11.444 ao final. Numa operação matemática simplória, verificamos que 19,48% do total de inscritos, foram aprovados. Mas lembrando que 59.832 bacharéis se inscreveram, pagando R$ 145,00 de taxa de inscrição, podemos depreender que a arrecadação total foi de R$ 8.675.640,00, isso apenas no primeiro exame de 2009, de um total de três exames. Forçoso concluir que no ano de 2009 a arrecadação da OAB com o exame de ordem venha a ultrapassar os 25 milhões de reais. É ou não uma mina de ouro!

Em entrevista ao O DIA[13], em 06.10.09, o presidente da OAB-RJ, Wadih Damous declarou que:
"A responsabilidade da Ordem dos Advogados está na identificação dos aptos a iniciar suas carreiras com um mínimo conteúdo necessário para enfrentar as dificuldades de seus clientes. O Exame não é insuperável. Longe disso, ajuda a fortalecer a categoria. A prova é que cerca de 7 mil novos advogados ingressam no mercado do Rio de Janeiro a cada ano."
Ora, essa conta deve ser contestada, a uma porque a OAB não tem competência para avaliar a qualidade das Instituições de Ensino Superior, no país, a duas porque, vejamos que no exame de ordem 39, o primeiro de 2009, a OAB-RJ teve um total de 7008 candidatos inscritos, desses apenas 1.795 foram aprovados ao final, num percentual de 25,61%, mantida essa proporção, chegamos a conclusão que em 2009 teremos, uma média, de 21.024 inscritos para 5.385 aprovados, bem menos que a declaração do presidente.

Não podemos negar a enorme existência de cursos jurídicos no Brasil, não podemos negar que alguns deles são deficientes, como acontece também com outras áreas do conhecimento superior, contudo, podemos afirmar que a OAB não tem competência para avaliar a qualidade do ensino, podemos afirmar que a qualificação profissional compete ao Poder Público, através do MEC. Podemos sim afirmar que é obrigação da OAB, como entidade de classe, a fiscalização dos advogados no exercício da advocacia, assim como procedem o CRM, o CRO, o CREA e assim por diante. Mas deixamos a sua conta as conclusões acerca do embate.
Bem sabemos que o ensino no Brasil, seja fundamental, médio ou superior, há décadas é deficiente, e não somente para a carreira do Direito, mas para todas as carreiras, sejam médicos, professores, engenheiros, economistas, arquitetos, etc. O Estado não investe o suficiente em políticas educacionais, não há material adequado, e os professores não são bem preparados, uma vez que, neste país não se sobrevive com os baixos salários, obrigando os docentes a terem três ou mais empregos, para sustentar a si e sua família. Este não é um privilégio dos docentes, a classe médica enfrenta o mesmo problema.

Basta para isso que comparemos o nosso sistema de ensino com o de outros países, tais como, Portugal, Finlândia, Suíça, Suécia, entre tantos outros, onde o investimento em educação e saúde é enorme, diga-se, investe-se na base de construção de uma nação. Segundo Cristovão Buarque[14], Senador e professor da Universidade de Brasília, o último censo escolar mostra uma tragédia hemorrágica no organismo do Brasil e a continuar assim uma anemia intelectual, como se transcreve:
Os dados do último censo escolar mostram a tragédia de uma hemorragia no organismo do Brasil: a redução no número de jovens que buscam a carreira do magistério nas nossas universidades. A continuar nesse rumo, o Brasil terá o agravamento da anemia intelectual que nos caracteriza. Em um mundo competitivo, isso significa a anemia na economia, na cultura, na vida social. Sem uma boa educação de base, não teremos uma boa universidade, porque desperdiçaremos os cérebros excluídos por falta de boa qualidade nos primeiros anos de educação. Mas sem uma boa universidade, não teremos boa educação de base, por falta de bons professores; esse é o círculo vicioso da hemorragia intelectual do Brasil.
A culpa está na falta de prestígio da carreira do magistério, por causa dos baixos salários, das vergonhosas condições de trabalho, da violência a que são submetidos os professores e da falta de adaptação da escola atual às necessidades e gostos das novas gerações. Mas a culpa está também na estrutura universitária, que não produz os profissionais de que o país precisa, nem com a qualificação necessária.
A universidade deve formar uma elite intelectual que se ponha a serviço do país, da população e da Humanidade.
Então modestamente apresentamos uma proposta a questão, a nosso sentir amplamente viável de ser aplicada, e no sentido de promover a educação do nosso país.
Uma vez que a sociedade tem a consciência de que o ensino no Brasil, em especial o Superior, é deficitário e forma profissionais de baixa qualidade, como dizem por aí, seria então por demais vantajoso para a sociedade, e principalmente para os profissionais brasileiros, que o MEC promovesse um exame nacional de conclusão do curso de graduação, sendo aplicado a todas as carreiras.

Seria muito simples, uma substituição do que é hoje o ENADE[15], mas, obrigatório para todas os cursos superiores e, agora a questão principal, como condição de obtenção do diploma de nível superior, a ser aplicado no último período do curso de graduação. Desta forma, o graduando que não atingisse o grau mínimo para aprovação, não poderia se formar, não obteria o diploma, devendo então prestar o próximo exame nacional.
E como sabemos, a maioria dos cursos de graduação, tem seus currículos divididos em períodos semestrais, portanto, o MEC poderia efetuar duas provas nacionais por ano, uma em julho e outra em janeiro. E claro, mediante o pagamento de uma taxa simbólica, que cobrisse os custos do exame nacional, não de forma a se tornar uma fonte de arrecadação, como é o caso do exame de ordem promovido pela OAB. Dessa forma ganharia a sociedade e os profissionais dos mais variados ramos do conhecimento, ganharia o Brasil.

Por fim, e de muita necessidade, a criação pelo MEC, da Licenciatura em Direito. A graduação de direito teria um ano a mais de duração para formar, para aqueles que assim o desejarem, professores de direito, pois como sabemos não existe curso de licenciatura plena em Direito. O que vemos no Brasil são magistrados, promotores, defensores, advogados, entre tantos outros, atuando também como professores, mas sem a devida qualificação, como é o caso dos que abraçam o magistério, e que tem um curso superior de licenciatura plena.

Por tudo que aqui foi exposto, cristalino é que o exame de ordem aplicado pela OAB é inconstitucional. Fere princípios constitucionais, dentre eles da igualdade, da isonomia, do direito ao exercício profissional e da legalidade e da dignidade da pessoa humana.

A OAB não tem competência para a qualificação profissional do bacharel de Direito, esta é, por definição legal, do Ministério da Educação e Cultura, conforme prevê a Lei de Diretrizes e Bases da Educação. Numa clara invasão de competência a OAB tenta usurpar essa função, sob o pretexto de que há uma enorme proliferação de cursos jurídicos no país, e ainda que os cursos existentes formam bacharéis que não estão aptos ao exercício profissional da advocacia.

Como demonstramos o exame de ordem da OAB é tanto formalmente, quanto materialmente inconstitucional. E ainda imoral, sob o aspecto arrecadatório, fomentando a vultosa arrecadação, sob a forma de taxa de inscrição, e ainda fomentando a proliferação de cursinhos, de formação duvidosa, tendo como, única e exclusivamente, a finalidade de aprovação nesse infame exame de ordem.
E por fim um questionamento para dar início a várias reflexões. Porque, tamanho alarde faz a OAB a favor do exame de ordem, se para ser Ministro do Supremo Tribunal Federal basta ter notável saber jurídico, reputação ilibada, ser indicado e aprovado em sabatina no Senado Federal? Esta é a previsão do art. 101 da Constituição Federal. Não precisa ser juiz, promotor, defensor público e nem advogado, basta ter, novamente afirmamos, notável saber jurídico, o qual a nossa Magna Carta não define.
Art. 101. O Supremo Tribunal Federal compõe-se de onze Ministros, escolhidos dentre cidadãos com mais de trinta e cinco e menos de sessenta e cinco anos de idade, de notável saber jurídico e reputação ilibada.
Parágrafo único. Os Ministros do Supremo Tribunal Federal serão nomeados pelo Presidente da República, depois de aprovada a escolha pela maioria absoluta do Senado Federal.
O presente artigo não se tem a pretensão em esgotar o assunto, mas esperamos ter contribuído para que a sociedade se esclareça mais sobre a questão. E é também uma contribuição ao apontar uma solução modesta para os cursos de graduação, formador de profissionais, apontados, por muitos, como deficitários.
[1] CONSULTOR JURÍDICO. Disponível em: http://www.conjur.com.br/2009-mar-06/juiza-permite-bachareis-tornem-advogados-passar-exame-oab. Acesso em: 10 jan 2010.
[2] BRASIL, Constituição da República Federativa do Brasil. Disponível em: http://www.planalto.gov.br/leg.asp. Acesso em: 10 jan 2010.
[3] MENDES, Gilmar Ferreira, Controle de Constitucionalidade incidental. Disponível em www.gilmarmendes.com.br. Acesso em 10 jan 2010.
[4] DIDIER JR., Fredie. Transformações do Recurso Extraordinário. In: Processo e Constituição. Estudos em homenagem a professor José Carlos Barbosa Moreira. Luiz Fux, Nelson Nery Júnior, Teresa Arruda Alvim Wambier (coordenadores). São Paulo: RT, 2006. Pág. 122.
[5] Supremo Tribunal Federal. Repercussão Geral. Disponível em: http://www.stf.jus.br/portal/glossario/verVerbete.asp?letra=R&id=451. Acesso em: 12 jan 2010.
[6] STF. Pronunciamento do Min. Marco Aurélio no RE 603.583-RS. Disponível em: http://www.stf.jus.br/portal/jurisprudenciaRepercussaoGeral/pronunciamento.asp?pronunciamento=3262858. Acesso em: 12 jan 2010.
[7] LIMA, Fernando Machado da Silva. Exame da OAB: repercussão geral. Disponível em: http://jus2.uol.com.br/doutrina/texto.asp?id=14251. Acesso em 27 jan 2010.
[8] BRASIL, Estatuto da Advocacia e a Ordem dos Advogados do Brasil. Disponível em: http://www.planalto.gov.br/ccivil_03/Leis/L8906.htm. Acesso em: 12 jan 2010.
[9] BRASIL. Lei n. 9.394/96 (Lei de Diretrizes e Bases da Educação). Disponível em: http://www.planalto.gov.br/ccivil_03/LEIS/l9394.htm. Acesso em: 12 jan 2010.
[10] BRASIL, Constituição da República Federativa do Brasil. Op. cit.
[11] OAB-MS. Damous prega defesa pública do Exame de Ordem pela valorização da advocacia no Brasil. Disponível em: http://www.jusbrasil.com.br/noticias/1639338/damous-prega-defesa-publica-do-exame-de-ordem-pela-valorizacao-da-advocacia-no-brasil. Acesso em: 12 jan 2010.
[12] OAB-RJ. Estatísticas. Disponível em: http://exo.oab-rj.org.br/index.jsp?conteudo=3817. Acesso em: 12 jan 2010.
[13] ODIA. Wadih Damous: Exame de Ordem. Disponível em: http://odia.terra.com.br/portal/conexaoleitor/html/2009/10/wadih_damous_exame_de_ordem_38985.html. Acesso em: 12 jan 2010.
[14] BUARQUE, Cristovam Ricardo Cavalcanti. Universidade do Magistério. Disponível em: http://www.cristovam.org.br/portal2/index.php?option=com_myblog&show=Universidade-do-magistA-rio-.html&Itemid=100113. Acesso em: 12 jan 2010.
[15] INEP. ENADE. O Exame Nacional de Desempenho de Estudantes (Enade), que integra o Sistema Nacional de Avaliação da Educação Superior (Sinaes), tem o objetivo de aferir o rendimento dos alunos dos cursos de graduação em relação aos conteúdos programáticos, suas habilidades e competências. O Enade é realizado por amostragem e a participação no Exame constará no histórico escolar do estudante ou, quando for o caso, sua dispensa pelo MEC. O Inep/MEC constitui a amostra dos participantes a partir da inscrição, na própria instituição de ensino superior, dos alunos habilitados a fazer a prova. Disponível em: http://www.inep.gov.br/superior/enade/enade_oquee.htm. Acesso em: 15 jan 2010.
[16] BRASIL, Constituição da República Federativa do Brasil. Op. cit.
BRASIL, Constituição da República Federativa do Brasil. Disponível em: http://www.planalto.gov.br/leg.asp. Acesso em: 10 jan 2010.
______, Constituição da República Federativa do Brasil. Op. cit.
______, Estatuto da Advocacia e a Ordem dos Advogados do Brasil. Disponível em: http://www.planalto.gov.br/ccivil_03/Leis/L8906.htm. Acesso em: 12 jan 2010.
______, Lei n. 9.394/96 (Lei de Diretrizes e Bases da Educação). Disponível em: http://www.planalto.gov.br/ccivil_03/LEIS/l9394.htm. Acesso em: 12 jan 2010.
BUARQUE, Cristovam Ricardo Cavalcanti. Universidade do Magistério. Disponível em: http://www.cristovam.org.br/portal2/index.php?option=com_myblog&show=Universidade-do-magistA-rio-.html&Itemid=100113. Acesso em: 12 jan 2010.
CONSULTOR JURÍDICO. Disponível em: http://www.conjur.com.br/2009-mar-06/juiza-permite-bachareis-tornem-advogados-passar-exame-oab. Acesso em: 10 jan 2010.
DIDIER JR., Fredie. Transformações do Recurso Extraordinário. In: Processo e Constituição. Estudos em homenagem a professor José Carlos Barbosa Moreira. Luiz Fux, Nelson Nery Júnior, Teresa Arruda Alvim Wambier (coordenadores). São Paulo: RT, 2006. Pág. 122.
INEP. ENADE. Disponível em: http://www.inep.gov.br/superior/enade/enade_oquee.htm. Acesso em: 15 jan 2010.
LIMA, Fernando Machado da Silva. Exame da OAB: repercussão geral. Disponível em: http://jus2.uol.com.br/doutrina/texto.asp?id=14251. Acesso em 27 jan 2010.
MENDES, Gilmar Ferreira, Controle de Constitucionalidade incidental. Disponível em www.gilmarmendes.com.br. Acesso em 10 jan 2010.
OAB-MS. Damous prega defesa pública do Exame de Ordem pela valorização da advocacia no Brasil. Disponível em: http://www.jusbrasil.com.br/noticias/1639338/damous-prega-defesa-publica-do-exame-de-ordem-pela-valorizacao-da-advocacia-no-brasil. Acesso em: 12 jan 2010.
OAB-RJ. Estatísticas. Disponível em: http://exo.oab-rj.org.br/index.jsp?conteudo=3817. Acesso em: 12 jan 2010.
ODIA. Wadih Damous: Exame de Ordem. Disponível em: http://odia.terra.com.br/portal/conexaoleitor/html/2009/10/wadih_damous_exame_de_ordem_38985.html. Acesso em: 12 jan 2010.
Supremo Tribunal Federal. Repercussão Geral. Disponível em: http://www.stf.jus.br/portal/glossario/verVerbete.asp?letra=R&id=451. Acesso em: 12 jan 2010.
______. Pronunciamento do Min. Marco Aurélio no RE 603.583-RS. Disponível em: http://www.stf.jus.br/portal/jurisprudenciaRepercussaoGeral/pronunciamento.asp?pronunciamento=3262858. Acesso em: 12 jan 2010.
CARLOS ALBERTO FERREIRA PINTO
Bacharel em Direito pela Universidade Estácio de Sá. Pós-graduado em Direito Civil, Processual Civil e Direito Empresarial pela FESUDEPERJ (Fundação Escola Superior da Defensoria Pública-RJ). Pós-graduado em Direito Penal e Direito Processual Penal pela Universidade Cândido Mendes. Pós-graduado em Direito Público e Tributário pela Universidade Cândido Mendes.
CARLOS ALBERTO FERREIRA PINTO
Bacharel em Direito pela Universidade Estácio de Sá. Pós-graduado em Direito Civil, Processual Civil e Direito Empresarial pela FESUDEPERJ (Fundação Escola Superior da Defensoria Pública-RJ). Pós-graduado em Direito Penal e Direito Processual Penal pela Universidade Cândido Mendes. Pós-graduado em Direito Público e Tributário pela Universidade Cândido Mendes.

segunda-feira, 24 de maio de 2010

É válida a investigação criminal direta do Ministério Público no contexto do Sistema Acusatório?

Como sabido, o constituinte brasileiro optou pela adoção do sistema acusatório em nosso ordenamento jurídico que, mesmo não vindo expressamente dito na Carta Magna, verifica-se a opção pelo modelo através da leitura do artigo 129, inciso I de seu texto legal, ao estabelecer ao Ministério Público o exercício da ação penal.
Ademais, observam-se princípios e garantias de ordem democrática insculpidos na Constituição, como o devido processo legal, imparcialidade do julgador, ampla defesa, contraditório etc, de modo a tornar mais garantista a atuação do processo penal, que deve limitar e legitimar o poder Estatal de punir, isto é, tornar viável e equilibrada a aplicação de pena bem como ser um efetivo instrumento de garantia dos direitos e liberdades individuais, fruto de grandes conquistas do homem, para proteger o cidadão dos poderes do Estado.
Tudo isso implica um modelo de Estado Democrático de Direito, que aponta como elemento principal a dignidade da pessoa humana.
Diante de tais argumentos, surge uma questão que vêm dividindo doutrina e jurisprudência no cenário atual: se é válida a investigação criminal direta do Ministério Público no contexto do Sistema Acusatório.
A resposta deve ser “não”. Não é válida a atuação do Ministério Público diretamente na persecução criminal, pelo menos em face do sistema acusatório, que rege nosso processo penal.
Vislumbra-se que tal argumento se impõe porque cabe ao Ministério Público o controle externo da atividade policial - “o que não significa a substituição da presidência da investigação, conferida ao delegado de carreira” [1] -, sendo esta uma forma de assegurar o sistema acusatório, onde cada um cumpre seu papel, sua função, sem adentrar na esfera de atribuições do outro e contaminar – se não o processo – sua própria consciência com um pré-julgamento do caso.
Deve o promotor, portanto, acompanhar as investigações e requisitar diligências quando necessário, jamais avocando para si a responsabilidade de investigar diretamente, apurando um crime; deve sempre fazê-lo de longe, apenas fiscalizando a atividade policial e interferindo somente para solicitar alguma diligência, assegurando, assim, que o acusado terá seus direitos resguardados.
Nesse contexto, Aury Lopes Jr.[2] aduz que:
[...] por falta de uma norma que satisfatoriamente defina o chamado controle externo da atividade policial [...], não podemos afirmar que o Ministério Público pode assumir o mando do inquérito policial, mas sim participar ativamente, requerendo diligências e acompanhando a atividade policial.

Caso o membro do Parquet realizasse diretamente a investigação, quem lhe fiscalizaria e quem garantiria os direitos do acusado? Haveria plena desigualdade das partes, tendo o juiz que intervir diretamente no inquérito para controlar a atividade investigativa do promotor, desvirtuando totalmente o sistema acusatório. O Poder Judiciário estaria fazendo as vezes do Ministério Público, entrando em contato com os autos antes da fase processual, o que poderia interferir no julgamento do magistrado. [3]
Portanto, o certo é que a Constituição atribui ao membro do Ministério Público a função de exercer o controle externo da atividade policial, e não o de substituí-la.
Ademais, não devem ser tidos como válidos e legítimos argumentos tais como “quem pode o mais, pode o menos” (teoria dos poderes implícitos) pelo qual um órgão com maiores poderes/atribuições pode exercer uma atividade de menor expressão, ou seja, muitos entendem que se cabe ao Ministério Público promover a ação penal, estaria ele legitimado a realizar diretamente a investigação. Contudo, estas são competências diversas, em que a Constituição se encarregou de explicitar quais seus titulares.
Outro ponto que merece atenção é o fato de o inquérito policial ser instrumento meramente facultativo e dispensável para propositura da ação penal. Em que pese esse entendimento, impossível aproveitar-se disso para afirmar que o Ministério Público realize a investigação. Não há a figura do promotor-inquisidor em nosso ordenamento jurídico.
De mais a mais, a Constituição Federal já se encarregou de atribuir à polícia a apuração de infrações penais, como dispõe o artigo 144, § 1º, incisos I e IV, e § 4º, da Constituição Federal, sendo forma da garantia constitucional do devido processo legal. [4]
Por derradeiro, tendo em vista a adoção do sistema acusatório, típico de um Estado Democrático de Direito, não deve ser tida como correta a interpretação da possibilidade de o Ministério Público investigar diretamente um crime, haja vista que sua presença na condução do inquérito é apenas “secundária, acessória e contingente”,[5] eis que aludida tarefa cabe à polícia judiciária.
Além disso, como afirma Nucci, “o sistema processual penal foi elaborado para apresentar-se equilibrado e harmônico, não devendo existir qualquer instituição superpoderosa”.[6] Seria uma quebra do garantismo, demonstrando nítido retrocesso; inconcebível uma instituição que zela pelos direitos e garantias fundamentais agir ilimitadamente na produção de provas para, posteriormente, per se, acusar.

Referências:
NUCCI, Guilherme de Souza. Código de processo penal comentado. 9. ed. São Paulo: Revista dos Tribunais, 2009.
LOPES JR., Aury. Direito processual penal e sua conformidade constitucional. 5. ed. Vol. I. Rio de Janeiro: Lumen Juris, 2010.




[1] NUCCI, Guilherme de Souza. Código de processo penal comentado. 9. ed. São Paulo: Revista dos Tribunais, 2009, p. 81.
[2] LOPES JR., Aury. Direito processual penal e sua conformidade constitucional. 5. ed. Vol. I. Rio de Janeiro: Lumen Juris, 2010, p. 259.
[3] “Ao transformar a investigação preliminar numa via de mão única, está-se acentuando a desigualdade das futuras partes, com graves prejuízos para o sujeito passivo. É convertê-la em uma simples e unilateral preparação da acusação, uma atividade minimalista e reprovável, com inequívocos prejuízos para a defesa.” LOPES JR., Aury. Direito processual penal e sua conformidade constitucional. 5. ed. Vol. I. Rio de Janeiro: Lumen Juris, 2010, p. 249.


[4] “Note-se, ainda, que o art. 129, III, da Constituição Federal, prevê a possibilidade do promotor elaborar inquérito civil, mas jamais inquérito policial.” NUCCI, Guilherme de Souza. Código de processo penal comentado. 9. ed. São Paulo: Revista dos Tribunais, 2009, p. 81.
[5]LOPES JR., Aury. Direito processual penal e sua conformidade constitucional. 5. ed. Vol. I. Rio de Janeiro: Lumen Juris, 2010, p. 260.
[6] NUCCI, Guilherme de Souza. Código de processo penal comentado. 9. ed. São Paulo: Revista dos Tribunais, 2009, p. 81.
Felipe Pinto Bruno - Advogado inscrito na OAB/DF, Pós-graduando em Ciências Penais pelo curso LFG - Brasília.

quarta-feira, 5 de maio de 2010

Trabalhador Vencido tem Direito de Recorrer sem Depósito Recursal, extendendo o Benefício da Gratuidade de Justiça

INEXIGIBILIDADE DE DEPÓSITO RECURSAL DO BENEFICIÁRIO DE ASSISTÊNCIA JUDICIÁRIA PARA EFEITO DE INTERPOSIÇÃO DE RECURSO ORDINÁRIO. EFETIVAÇÃO DAS GARANTIAS PROCESSUAIS CONSTITUCIONAIS E DOS DIREITOS E GARANTIAS PROCESSUAIS CONSTITUCIONAIS E DOS DIREITOS E GARANTIAS FUNDAMENTAIS DO CIDADÃO, CONSUBISTANCIADOS NO PRINCÍPIO DO DEVIDO PROCESSO LEGAL. LIMITE DO DUPLO GRAU DE JURISDIÇÃO. Advindo condenação pecuniária para o trabalhador beneficiário de assitência judiciária, inexigível o depósito recursal de que trata o §1º, do art. 899/CLT, porquanto nos moldes do inciso VII, do art. 3º, da Lei 1.060/50, acrescido pela LC 132/09, a assistência judiciária compreende a ausência de recolhimento dos depósitos previstos em lei para a interposição de recurso, como meio de exercício dos direitos e garantias fundamentais do cidadão, consubstanciados no inciso LV, do art. 5º/CF, que assegura aos litigantes o contraditório e a ampla defesa, com os meios e recursos a ela inerentes, rol em que se insere o duplo grau de jurisdição, cuja atuação se materializa na interposição de recursos das decisões de origem. (TRT/3 - 3ª T., Processo: 00034-2008-084-03-40-1- AIRO, Rel. Juiz Vitor Salino de Moura Eça, Publicação do DEJT de 19/04/2010).
Tem-se portanto, talvez de forma inédita o direito do trabalhador vencido recorrer sem o depósito recursal, extendendo o benefício da justiça gratuita.
Colaboração: Vitor Salino de Moura Eça - Juiz do Trabalho no Tribunal Regional do Trabalho da 3ª Região - Minas Gerais; Professor de Direito Processual do Trabalho e Direito Processual do Trabalho Comparado nos cursos de graduação e pós-graduação em Direito da PUC-MG e da Escola Judicial do TRT/3; Doutor em Direito Processual e Mestre em Direito do Trabalho pela PUC-MG; Especialista em Direito do Trabalho pela Universidad de Buenos Aires - Argentina e em Direito Empresarial pela UGF/RJ. Membro efetivo de diversas associações, dentre elas: Associación Iberoamericana de Derecho del Trabajo y de la Seguridad Social - AIDTSS; da Associação Latino-Americana de Juízes do Trabalho - ALJT; do Equipo Federal del Trabajo - RFT, e do Instituto Brasileiro de Direito Social Cesarino Júnior - IBDSCJ - Seção brasileira da Société Internacionale de Droit du Travail el de la Sécurité Sociale - SIDTSS

terça-feira, 26 de janeiro de 2010

As Alterações Introduzidas pela Lei n.º 11.719 de 20 de junho de 2008 no Processo Penal

Considerações Iniciais
O presente trabalho tem como proposta estudar as inovações introduzidas pela lei n.º 11.719 de 20 de junho de 2008 no Código de Processo Penal, no que diz respeito à atuação do juiz de ofício, quando da prolatação de sentença condenatória, e, principalmente, quando nesta, há a nova definição do fato típico, ou seja, do crime, mesmo que o novo enquadramento seja em crime mais grave, ou sejam reconhecidas circunstâncias agravantes ao crime.
O Código de Processo Penal, antes da alteração promovida pela lei n.º 11.719 de 20 de junho de 2008, dava liberdade ao juiz, de definir juridicamente o fato de forma diversa da que constasse na queixa ou denúncia, ainda que essa nova definição do fato tivesse sanção mais grave, devendo apenas, baixar o processo para que, a defesa manisfeste-se em 8 (oito) dias, e caso queira, produza provas, podendo ser ouvidas até 3 (três) testemunhas. Em se tratando de ação penal pública o Ministério Público aditasse a denúncia, e posteriormente abrindo-se prazo de 3 (três) dias à defesa para que esta oferecesse provas, podendo arrolar até 3 testemunhas.
A lei n.º 11.719 de 20 de junho de 2008 altera os dispositivos legais relativos à suspensão do processo, emendatio libelli, mutatio libelli e aos procedimentos constantes do Código de Processo Penal, modificou a faculdade que o juiz tinha de aplicar e/ou reconhecer de ofício as circunstâncias agravantes do crime objeto do processo penal em curso, limitando sua atuação apenas a aplicação da lei em si, sem modificação da descrição do fato típico em sim; e, observando também, a partir dessa nova definição, se há a possibilidade de suspensão condicional do processo, bem como se é competente para julgar tal fato. No caso de ser possível a suspensão do processo, o juiz procederá de acordo com o disposto em lei; e, no caso de incompetência de juízo, esse deverá encaminhar os autos ao juízo competente para julgar a infração, dando assim maior celeridade ao processo.
Em As Alterações introduzidas pela lei n.º 11.719 de 20 de junho de 2008 no processo penal, visa-se apresentar as questões mais importantes sobre o emendatio libelli, mutatio libelli, suspensão e procedimentos processuais.
Dentre as questões mais relevantes a serem abordadas na análise do tema em foco, pode-se enumerar as seguintes indagações:
Quais as principais alterações introduzidas pela lei n.º 11.719 de 20 de junho de 2008, no que diz respeito à sentença e atuação do juiz?
Como era, e agora como ficou a operacionalidade do instituto do emendatio libelli?
Como era, e agora como ficou a operacionalidade do instituto do mutatio libelli?
É possível a atuação do juiz, de ofício, na celeridade processual no que diz respeito à absolvição com o advento da lei n.º 11.719/2008?
Temos, portanto, como objetivo a demonstração da reforma processual penal de 2008, expondo os poderes da figura do juiz, suas limitações após a lei n.º 11.719 de 20 de junho de 2008, sua função nos institutos de emendatio libelli e mutatio libelli, bem como a atuação do Ministério Público também nesses institutos.
Objetiva-se também a exposição da atuação do juiz na aplicação de agravantes, em sentença condenatória, mesmo quando o Ministério Público tenha opinado pela absolvição do réu, levando em consideração os preceitos fundamentais inseridos na Carta Magna, que, de forma garantista, vai de encontro ao Código de 1940, época em que se imperava no país, e na sociedade de modo geral, um modo autoritário de governo e pensamento.
Sendo a escolha do presente tema motivada por sua importância em relação à atuação do juiz e do Ministério Público no processo penal, quando da denúncia e seu recebimento, seu aditamento quando necessário, sua análise, tese defensiva, e finalmente, sua sentença.
Há quem sustente que os artigos que davam a total liberdade ao juiz de dar ao fato definição diversa da contida nos autos, de forma a observar os agravantes do fato, e consequentemente o aumento de pena a ser cumprida pelo réu, como preceituado no antigo artigo 383 feria o princípio da correlação entre imputação e sentença, que é uma das principais garantias do direito de defesa, e tutelado por via constitucional.
Outros sustentam até que esse artigo não deveria sequer ter sido recepcionado pela Carta Magna, uma vez que o Código de Processo Penal é anterior à promulgação da Constituição Federal, afinal, o Código foi elaborado em pleno Estado-Novo.
Ocorre que esse Código tem de aproximadamente 68 (sessenta e oito) anos de existência, e, passado por 3 (três) Constituições, sendo a última, a de 1988, a mais democrática e garantista de todas as outras, e por via de conseqüência com algumas divergências e confrontos com o já idoso Código.

- Principais alterações introduzidas pela lei n.º 11.719 de 20 de junho de 2008, no que diz respeito à sentença e atuação do juiz:
A lei n.º 11.719/2008 introduziu algumas modificações no processo penal em si, principalmente na atuação de ofício do juiz.
A primeira modificação foi em relação ao artigo 383 do Código de Processo Penal, que antes da lei, dava liberdade ao juiz de dar nova definição jurídica ao fato, mesmo que essa nova definição fosse mais grave que a contida na denúncia ou queixa, além de reconhecer agravantes, caso existissem.
Com a lei n.º 11.719/2008, que lhe acrescentou 2 (dois) parágrafos, essa liberdade deixou de ter seu caráter incisivo, e indicou ao julgador que, mesmo dando definição jurídica ao fato diversa da contida na denúncia ou queixa, sem, é claro, modificar a descrição do fato, ele, de ofício, em conseqüência dessa nova definição, deverá proceder o disposto em lei no que tange a suspensão condicional do processo; desde que, caiba ao caso concreto. E ainda, com essa nova definição, se o juízo não for ou deixar de ser competente, este, deverá, também de ofício, encaminhar os autos para o juízo competente.
Outra modificação introduzida pela lei 11.719/2008 foi em relação ao artigo 384 do mesmo diploma legal que retirou o parágrafo único, e acrescentou 5 (cinco) parágrafos. Onde anteriormente se previa que o juiz, ao reconhecer a possibilidade de nova definição jurídica ao fato, baseado em provas existentes nos autos, circunstâncias elementares explícitas ou implícitas na denúncia ou na queixa, deveria baixar o processo, para que a defesa se manifestasse em 8 (oito) dias, e se quisesse, produzisse prova, podendo ser ouvidas até 3 (três) testemunhas. No caso da possibilidade de nova definição jurídica do fato, se essa importasse em aplicação de pena mais grave, o juiz deveria baixar o processo afim de que o Ministério Público pudesse aditar a denúncia ou queixa, nos casos de ação penal pública, e, em seguida, a defesa tinha 3 (três) dias para oferecer prova, arrolando até 3 (três) testemunhas.
Após a referida lei, esse procedimento ganhou maior celeridade, pois, logo após o encerramento da instrução probatória, poderá ocorrer o aditamento da denúncia ou queixa, quando desta houver a instauração de ação pública, que deverá ser feito em 5 (cinco) pelo Ministério Público, que, caso não o faça, os autos serão remetidos ao Procurador-Geral para que o faça, ou designe outro promotor para fazê-lo, e, só então, se estes não entenderem pelo aditamento, ou entenderem pelo arquivamento, aí sim, o juiz será obrigado a atender a opinião do Ministério Público.
Assim, se deverá ser ouvida a defesa no prazo de 5 (cinco) dias, e, se admitido o aditamento, o juiz, a requerimento das partes designará audiência com inquirição de testemunhas, novo interrogatório do acusado, realização de debates e julgamento, podendo ser arroladas em 5 (cinco) dias 3 (três) testemunhas para cada parte, devendo o juiz ficar, na sentença, adstrito aos termos do aditamento, afinal, o processo deve estar já saneado,o que vem a imprimir maior celeridade à aplicação da tutela jurisdicional penal.
Caso não seja recebido o aditamento, o processo prosseguirá seguindo os mandamentos do Código de Processo Penal.

- A operacionalidade do instituto do emendatio libelli:
Antes do advento da lei n.º 11.719/2008, o instituto do emendatio libelli consistia em, o juiz verificando a comprovação dos fatos, e as circunstâncias constantes do processo, podia condenar o acusado, dando ao delito a definição jurídica que entendesse cabível e não a articulada na peça inicial, podendo até, em sentença, condenar o réu por outro crime descrito, ou seja, sem que houvesse a específica imputação.
Atualmente, com a referida lei, esse poder do juiz, deixou de ser absoluto, uma vez que lhe é dada a faculdade atribuir definição jurídica diversa à dos autos, ainda que essa seja mais grave, desde que não modifique a descrição do fato, mas, caso essa nova definição possibilite a suspensão condicional do processo, o juiz deverá proceder de acordo com o disposto em lei, e aplicável ao caso.
Se a nova definição jurídica do fato deixar de ser de competência do juízo que a proferiu, deverá então, encaminhar ao juízo competente, imprimindo assim maior celeridade ao processo.
Na emendatio libelli não ocorre uma verdadeira alteração do libelo, e sim uma corrigenda da peça acusatória.

- A operacionalidade do instituto do mutatio libelli:
O mutatio libelli, é a verdadeira alteração do libelo, que ocorre durante a instrução do processo, onde se colhem provas de que existem elementos essenciais que estão contidos, expressa ou implicitamente na denúncia, de modo que a sentença não pode ser proferida de imediato.
Assim, anteriormente à lei n.º 11.719/2008, o processo era baixado para que a defesa se manifestasse, e se quisesse produzisse prova, podendo ser ouvida até 3 (três) testemunhas, no prazo de 8 (oito) dias, sob pena de nulidade.
Caso o juiz verificasse que deveria aplicar pena mais grave do que a resultante da imputação não contida expressa ou implicitamente na denúncia, deveria baixar o processo para que o Ministério Público aditasse a denúncia, e, posteriormente, manifestação da defesa, no prazo de 3 (três) dias, podendo inclusive arrolar 3 (três) testemunhas e produzir prova.
Já com o advento da já citada lei, logo depois de encerrada a instrução probatória, o Ministério Pública, caso entenda cabível, poderá aditar a denúncia num prazo de 5 (cinco) dias, podendo inclusive fazer oralmente.
Observa-se também que, caso o Ministério Público não entenda pelo aditamento, o juiz remete os autos ao Procurador-Geral para que esse o faça, ou designe novo membro do Ministério Público para fazê-lo, e, só assim, caso esse não entenda pelo aditamento, esse não será feito, ou ainda, se esse entender pelo arquivamento do processo, assim será feito.
Antes de se admitir o aditamento, a defesa deve ser ouvida em 5 (cinco) dias, e, admitido o aditamento, uma audiência será marcada para inquirição de testemunhas, novo interrogatório do acusado, realização de debates e julgamento, desde que requerida por qualquer das partes.
Se a partir do aditamento houver condição de suspensão condicional do processo, o juiz assim o fará, observando o procedimento disposto em lei; bem como, se deixar de ser competente, encaminhará ao juízo competente.
Em havendo o aditamento, cada parte poderá arrolar até 3 (três) testemunhas, no prazo de 5 (cinco) dias, ficando o juiz, na sentença, adstrito aos termos do aditamento. Caso não seja recebido o aditamento, o processo prosseguirá.

- A atuação do juiz, de ofício, na celeridade processual no que diz respeito à absolvição com o advento da lei n.º 11.719/2008:
No processo penal, o juiz age sempre em prol dos esclarecimentos da verdade real dos fatos, para dirimir qualquer dúvida que possa ainda existir nos autos do processo, formando, portanto, seu convencimento para julgar a causa.
O Código de Processo Penal não adotou o princípio da identidade física do juiz, princípio o qual o magistrado que não presidiu a instrução do processo, não pode prolatar sentença. Só poderia fazê-lo após determinar novamente toda a instrução.
Esses preceitos foram revogados pela lei n.º 11.719/2008, evitando assim que o processo penal se arraste, tomando mais tempo do que deveria.
Parte da doutrina defende, basicamente, a atuação do juiz, de ofício, só deve ser válida quando em benefício do réu, como prescrito na Lei Maior, considerando nula, ou, ao menos passível de anulação os atos que prejudiquem o réu, seja por não observarem o contraditório, seja por não observarem o devido processo legal.
De acordo com o artigo 385 do Código de Processo Penal, o juiz poderá proferir sentença condenatória, ainda que o Ministério Público tenha opinado pela absolvição, bem como reconhecer agravantes, embora nenhuma tenha sido alegada, nos casos de ação penal pública. Podem ser reconhecidas, inclusive, as circunstâncias não articuladas na denúncia, desde que presentes no fato típico.
Outra atuação de ofício do juiz é no sentido de absolver o réu (também foi modificada pela lei 11.719/2008), desde que mencione a causa na parte dispositiva reconhecendo estar provada a inexistência do fato, não haver prova da existência do fato, não constituir o fato infração penal, estar provado que o réu não concorreu para a infração penal, não existir prova de ter o réu concorrido para a infração penal, existirem circunstâncias que excluam o crime ou isentem o réu de pena, ou mesmo se houver fundada dúvida sobre sua existência, não existir prova suficiente para a condenação, demonstrando assim, a observância dos preceitos fundamentais contidos na Constituição da República, de forma visivelmente garantista.
Logo, sob esse ponto de vista, do garantismo constitucional, tornou a referida lei tão importante para o ordenamento jurídico penal, já que além de imprimir mais celeridade ao processo, faz os operadores do direito observarem e respeitarem as garantias fundamentais, que são os pilares da Constituição Federal de 1988.
Ainda na sentença absolutória, o juiz deverá mandar, se for o caso, pôr o réu em liberdade, ordenar a cessação das medidas cautelares e provisoriamente aplicadas, aplicar medida de segurança, caso cabível.
Conclui-se que, a alteração do Código de Processo Penal ao ‘tentar’ privilegiar a celeridade deixou de observar alguns preceitos constitucionais, como o do contraditório e ampla defesa, peças essenciais ao garantismo, reinante no ordenamento jurídico, principalmente em sua corte suprema.

Diego Costa
Advogado inscrito na OAB/RJ, pós-graduando em Direito Processual Civil

O Trabalho Como Direito Humano – Ensaios.

O trabalho é título de honra, que toca a dignidade humana, que o salário deve ser justo e suficiente para manter o trabalhador e sua família de forma decorosa (Manuel Alonso Garcia – Derecho del Trabajo, Barcelona, José Maria Bosch Editor. 1960, p. 122).
O trabalho tem sentido reconstrutivo e na visão hebraica, adquire uma valorização como atividade humana.
E finalmente no Renascimento podemos apontar como ápice do trabalho que é visto como verdadeira essência humana. (Felice Battaglia – Filosofia do Trabalho, São Paulo, Ed. Saraiva, p. 30).
Talvez todas essas idéias estejam misturadas no mundo contemporâneo e globalizado, já que o trabalho vem delineado em nossas vidas desde que nascemos. “trabalho de parto – trabalho que damos a nossas mães – e finalmente, o que vamos ser quando crescermos – (qual trabalho realizaremos para nosso sustento).
Então é neste pequeno intróito que podemos nos questionar: o trabalho seria um direito humano?
É claro que neste artigo –ensaio – não pretendemos traçar definições precisas ou responder definitivamente a questão.
Na evolução histórica do próprio sentido do trabalho na sociedade, temos como mais intrigante a sociedade grega.
Os mais bem sucedidos não trabalhavam verdadeiramente, pois o trabalho só cabia aos escravos. Os ricos precisavam estar livres do trabalho para se ocupar apenas intelecto-artístico.
Por sorte, ou azar, o homem e a sociedade evoluíram e nos presentearam com a tão famosa liberdade e igualdade. E fizeram com que, ao longo destes anos o trabalho passasse a fazer a parte da própria dignidade do ser humano.
Já de há muito está estabelecido no artigo 23 da Declaração Universal dos Direitos Humano”.destaca”, bem como no artigo 1º da Constituição da República Federativa do Brasil “destaca III e IV incisos”.
Adam Smith também já dizia que toda fonte de riqueza provém do trabalho.Esses direitos e garantias fundamentais devem ser exigidos e exercidos pelos cidadãos brasileiros, porque é o próprio artigo 1º que enuncia que o Estado precisa promover o trabalho.
Então o poder público, de um modo geral, não pode se omitir de cumprir esse direito subjetivo público, porque inviabiliza a própria dignidade humana.
A propósito, nos define Alexandre de Moraes in Direito Constitucional 14ª Edição, Ed. Atlas, p. 50: “dignidade da pessoa humana...é o mínimo invulnerável que todo estatuto jurídico deve assegurar de modo que somente excepcionalmente possam ser feitos limitações ao seu exercício...” “...trabalho – é através do trabalho que o homem garantir sua subsistência e o crescimento do país...”
Não seria demais pontuar que nesta concepção de Estado, como na ação política e juridicamente organizada comprometida num território com poder soberano e um povo livre, visando à garantia de vida deste, constatamos que uma das principais funções do Estado e dar ao povo a dignidade humana, que começa com o trabalho.
Não o assistencialismo, que vimos se perpetrar no nosso país; que apenas faz engrossar a massa dos miseráveis.Mas sim ver a nossa Carta Magna ser colocada na prática. O povo não pode ser tratado como coitado. “É preciso ensinar a pescar, e não dar o peixe; sem o devido esforço”.
A nossa Constituição não despreza os pilares dos direitos fundamentais, e baseia a sua ordem econômica na valorização do trabalho.
Modernamente classificamos esses direitos fundamentais em:

1ª geração - direitos e garantias individuais.
2ª geração – direitos sociais, econômicos e culturais
3ª geração – direitos de solidariedade e fraternidade.

Então não seria o direito ao trabalho, um direito a dignidade humana, ou seja, um direito humano?
Desde os primórdios os filósofos e juristas já tinham dificuldade para definir direitos humanos: - direitos e liberdades básicas dos seres humanos.
Por fim, para tentarmos concluir, nos ensina Norberto Bobbio, in Teoria Geral do Direito, p. 9. Editora Martins Fontes:

“os elementos constitutivos do conceito de direito são três: a sociedade como base de fato em que o direito passa a existir, a ordem como o fim a que atende o direito e a organização como meio para realizar a ordem...Isso significa que o direito nasce no momento em que um grupo social passa de uma fase não organizado para uma fase organizada. É o processo de institucionalização. Deste processo nascem as normas de conduta, que tem sua origem na lei Maior do País - a Constituição. As ações afirmativas deste contidas nesta Constituição, não podem apenas ser mecanismos de compensação, mas medidas de integração. Este é o constitucionalismo moderno.”

Todavia, não adianta prever o trabalho como a própria dignidade humana e não garanti-lo efetivamente, tentando justificar o não cumprimento da Constituição, por fatores econômicos mundiais, tecnológicos e até burocráticos.
A criação de postos de trabalho e a sua garantia é um direito inalienável do cidadão, mas não adianta pretender classificá-lo como direito humano, sem ir ao começo de tudo como bem destacado por Norberto Bobbio, a sociedade, a ordem e a organização .
E essa sociedade tem que visar E essa sociedade tem que visar antes de qualquer coisa a , educação, sem a qual o cidadão nunca estará apto nem a concorrer ao seu direito humano ao trabalho. Não se pode imaginar hoje, que o baixo nível de qualificação de nossos cidadãos possa lhes dar igualdade de condições na concorrência dos postos de trabalho que estão cada vez mais escassos.
O cidadão tem o direito ao trabalho digno, com respeito às legislações do trabalho e social, para que não viva à margem da sociedade, lutando, hoje, com um Estado que mais valoriza as corporações do que seus trabalhadores, esquecendo-se, que o trabalhador, antes de tudo, um ser humano é peça fundamental na engrenagem de uma nação desenvolvida.

A competência da Justiça do Trabalho na fase pré-contratual da relação de emprego.

Dispõe o artigo 114 da Constituição da Republica Federativa do Brasil:

Art. 114 - Compete à Justiça do Trabalho processar e julgar: (Alterado pela EC-000.045-2004)
I - as ações oriundas da relação de trabalho, abrangidos os entes de direito público externo e da administração pública direta e indireta da União, dos Estados, do Distrito Federal e dos Municípios;

“IX - outras controvérsias decorrentes da relação de trabalho, na forma da lei.”

Não há dúvida que para solucionar direitos e obrigações decorrentes das relações de trabalho, em sentido amplo. Numa situação hipotética, que muito tem ocorrido nos dias de hoje, de um concursado que esteja classificado em um concurso público, para emprego público, a ser regido pela CLT, em que o empregador público, pretere os classificados, fazendo contratos temporários com empresas terceirizadas.
Seria a Justiça do Trabalho competente para dirimir conflitos daí decorrentes?
Basta que o litígio derive de uma dessas modalidades de relação jurídica, (ainda que não formalizada), do trabalho, principalmente se o pedido a ser formulado se dirige a potencial empregador.
Já de algum tempo o STF, tem sedimentado entendimento de que a contratação pela Administração Pública, com base em relação jurídica contratual de emprego, isto é, celetista, é suficiente para determinar a competência material da Justiça do Trabalho para julgar pretensões do futuro empregado público, principalmente após o advento da Emenda 45/2004, que alterou o artigo 114 da CRFB.
Nos ensina Liebman que a competência se liga diretamente com a relação jurídica controvertida, se a relação jurídica material a ser discutida é de direito do trabalho, vê-se claro, num primeiro momento a competência da especializada.
Nos ensina Carlos Henrique Bezerra Leite, 4ª Edição, Curso de Direito Processual do Trabalho, p. 191 que “ no que concerne a relação empregatícia existente no âmbito das empresas públicas e sociedade de economia mista, a competência da Justiça do Trabalho está implícita no inciso I,, do artigo 114 da CF.(competência material original). Mesmo porque os servidores também chamados de “empregados públicos”, são aplicáveis todos os direitos materiais dos empregados em geral, tal como se depreende da nossa Carta no artigo 173, parágrafo 1º.”
Esse é o entendimento que vem sendo adotado também pelo TST:
PROCESSO: RR NÚMERO: 625355 ANO: 2000PUBLICAÇÃO: DJ - 01/07/2005
2.1. JUSTIÇA DO TRABALHO. COMPETÊNCIA MATERIALComo corolário do conhecimento recurso, por violação ao artigo 114 da Constituição Federal, dou provimento ao recurso de revista para anular o acórdão recorrido, em face de erro procedimental, e determinar a remessa dos autos ao TRT para prosseguir no julgamento do mérito do recurso ordinário dos Reclamantes, afastada a incompetência material da Justiça do Trabalho.ISTO POSTOACORDAM os Ministros da Primeira Turma do Tribunal Superior do Trabalho, unanimemente, conhecer do recurso de revista, por violação ao artigo 114 da Constituição Federal; no mérito, dar-lhe provimento para anular o acórdão recorrido, em face de erro procedimental, e determinar a remessa dos autos ao TRT para prosseguir no julgamento do mérito do recurso ordinário dos Reclamantes, afastada a incompetência material da Justiça do Trabalho. Ademais, a doutrina tem reconhecido a competência desta Justiça Especializada para apreciar e julgar os pedidos referentes ao pré-contrato, em especial quando o pedido deduzido diz respeito à não-convocação de aprovados em concurso público realizado por entidade de direito privado, integrante da Administração Pública direta ou indireta. A competência da Justiça do Trabalho, na hipótese, decorre da potencial e futura relação de emprego a se concretizar com a reclamada, consoante expressa o artigo 173, § 1º, II da CF/88. A pretensão obreira está, inexoravelmente, protegida pelo manto constitucional do artigo 114 da Carta Magna, para quem a Justiça do Trabalho é competente para processar e julgar outras controvérsias decorrentes do contrato de trabalho. Portanto, é pela natureza da relação jurídica substancial que se estabelece a distinção entre as diversas estruturas judiciárias, inexistindo restrições de cunho jurídico no reconhecimento da competência da Justiça Especializada, para julgar ação de índole pré-contratual. A egrégia 5ª Turma, examinando matéria assemelhada, nos autos do RR-702.000/00, da lavra do Juiz Convocado Walmir Oliveira da Costa, tendo como recorrente o Ministério Público da lavra do Juiz Convocado Walmir Oliveira da Costa, tendo como recorrente o Ministério Público do Trabalho, em que se discutia direitos situados em momento pré-contratual, firmou o seguinte entendimento: “Vale registrar, em última análise, que a Justiça do Trabalho também não é avessa ao deslinde daquelas lides em que a causa de pedir e o pedido tenham relação com prejuízos morais e materiais que possam advir das tratativas preliminares ao pacto de emprego, ou seja, na chamada fase pré-contratual, em que as partes reuniam as condições inerentes às figuras de empregado e empregador, mas a contratação não chegou a ser concretizada. Irrecusável, portanto, a competência material da Justiça do Trabalho para julgar a presente Ação Civil Pública. São, ainda, precedentes da Corte: RR-779.846/2001, de minha lavra, DJ de 21/03/03; RR-86054/2003, DJ de 02/04/04, RR-791324/01, DJ de 30/01/04, RR-698549/00, DJ de 20/06/03 e RR-446080/98, DJ de 09/02/01, Relator Min. Antônio Barros Levenhagen, 4ª Turma.Ante o exposto, intacto o artigo 114 da CF/88, não conheço do recurso de revista interposto.”(extraídos do site:www.tst.gov.br)

Que não se olvide que de há muito, o STF”., já havia entendido que: “Quando o Estado pratica atos jurídicos regulados pelo direito civil (comercial ou trabalhista) coloca-se no plano dos particulares” RDA 46/192, RTJ 29/465, 39/462,...”

Voltando a questão hipotética para melhor analisar a competência, não podemos nos olvidar que a administração pública direta ou indireta no exercício da atividade administrativa deve se nortear, pelos princípios da legalidade e moralidade.
É do inciso II do artigo 37 da CRFB:

“II- a investidura em cargo ou emprego público depende de aprovação prévia em concurso público de provas e títulos, de acordo com a natureza e a complexidade do cargo ou emprego na forma prevista em lei...”

Logo, ao editar um ato administrativo, in casu, edital a que se submete o concurso público, a administração pública o fez, destinando verbas em grande escala para a sua realização, fazendo–o motivadamente, fundado em sérias razões, e, principalmente na premente necessidade do provimento de cargos a que se destina o certame.
O pressuposto lógico de todo ato administrativo, como sabemos, deve ser também a boa-fé, lealdade e respeito aos administrados. O concurso público, entendido como conjunto de atos administrativos que visa à a aferição das aptidões de candidatos ao fito de selecionar os melhores para o provimento dos cargos públicos.
Pactua-se assim, normas preexistentes entre os dois sujeitos da relação editalícia: Administração e Administrado/candidato. Tal relação tem o poder de vincular ambas as partes
Gozam de presunção de legitimidade, uma vez editados contam com a prerrogativa: serem sempre voltados para o interesse público. Ora, se o candidato/ administrado cumpre com suas obrigações, i.e., estudando e sacrificando-se, gastando seu tempo e dinheiro na esperança de ser aprovado e contratado (como previsto no edital) e ter o verdadeiro retorno de seu investimento, a não contratação deste concursado, preterido por contratos temporários com empresas terceirizadas, implica na quebra do princípio da boa-fé, da lealdade, da moralidade, dentre outros acima mencionados.
Com efeito, segundo José dos Santos Carvalho Filho in Manual de Direito Administrativo. Editora Lumen Iuris, p. 19, 20ª. Edição, que “o princípio da moralidade está associado a “preceitos éticos” a distinguir o que honesto do que não é honesto, e cremos que ninguém de bom senso pode admitir ser ético, honesto, convocar um concurso público e não contratar os aprovados , mesmo existindo vagas para outros” empregados”. Se não pretendia substituir seus “terceirizados” porque realizou o concurso?
Isso seria gestão temerária, movimentar o aparato estatal sem o devido interesse público e deve ser coibido pelo Judiciário, in casu, do Trabalho. Estabelece o artigo 41, da Lei 8666/93 que a “Administração não pode descumprir as normas e condições do edital ao qual se acha estritamente vinculada”. Ora, nessas condições a não nomeação de candidato aprovado em concurso público dentro do quadro de vagas ou para o famoso “cadastro de reservas”, quando existem empregados “terceirizados” ocupando as supostas vagas, faz surgir a lesão, não mais a uma mera expectativa de direito e sim a um direito subjetivo, que deve ensejar a tutela jurisdicional.
Frise-se, por oportuno,que esta moralidade administrativa, comparada a boa-fé objetiva do direito privado, que tende sempre a atender a finalidade, que é requisito mor do ato administrativo.
Portanto se a finalidade (interesse coletivo/público), elemento vinculado do ato administrativo, in casu, preencher vagas e formar o cadastro de reserva para futuras vagas, se existem empregados terceirizados ocupando estas vagas, preterindo a chamada dos candidatos legitimamente aprovados, há no mínimo um abuso de direito por parte do Administrador, que tem que ser coibido pelo Judiciário.
Vale destacar o voto proferido por Hamilton Carvalhido e Helio Quaglia Barbosa em recente julgado do STJ:

“O candidato aprovado em concurso público tem mera expectativa de direito à nomeação, que deve ser praticada por conveniência e oportunidade da administração pública. Mas essa aprovação se torna direito subjetivo se houvesse inequívoca manifestação da necessidade de provimento do cargo durante o prazo de validade do concurso ou ainda que houvesse contratação de pessoal para preenchimento das vagas em flagrante desrespeito aqueles regularmente aprovados, e aptos a ocupar o mesmo cargo.”

“Concurso Público: Direito à Nomeação
Por ofensa ao art. 37, IV, da CF ("durante o prazo improrrogável previsto no edital de convocação, aquele aprovado em concurso público de provas ou de provas e títulos será convocado com prioridade sobre novos concursados para assumir cargo ou emprego, na carreira;"), a Turma deu provimento a recurso extraordinário para assegurar a nomeação de candidatos aprovados em concurso público para o cargo de professor assistente da Universidade de São Paulo - USP. Considerou-se que, no caso concreto, ficara comprovada a necessidade da Administração no preenchimento das vagas, haja vista que a Universidade de São Paulo contratara, no prazo de validade do concurso, dois professores para exercerem o mesmo cargo, sob o regime trabalhista - sendo um deles candidato aprovado do mesmo concurso. Afastou-se, ainda, a fundamentação constante do acórdão recorrido no sentido de que seria necessária a abertura de novo concurso pela Administração para a comprovação da existência das vagas. Precedente citado: RE 192.568-PI (DJU de 13.9.96).”
RE 273.605-SP, rel. Min. Néri da Silveira, 23.4.2002. (RE-273605)
MANDADO DE SEGURANÇA. CONCURSO PÚBLICO. DECADÊNCIA. PRAZO INICIAL. DATA DE EXPIRAÇÃO DO PRAZO DE VALIDADE DO CONCURSO. REVISÃO DE ATO ADMINISTRATIVO. CONTRATAÇÃO TEMPORÁRIA. PRETERIÇÃO DE CANDIDATO APROVADO. 1.Em se tratando de pedido de nomeação em cargo público mediante aprovação em concurso, o prazo decadencial começa a fluir da data de expiração da validade do concurso público. A Constituição Federal, no seu artigo 37, incisos II,IV e IX, a obrigatoriedade da realização de concurso público para o provimento de cargos e funções relativas às atividades típicas de Estado, excetuando as hipóteses de investidura em cargo em comissão e de contratação temporária para atender a excepcional interesse público. O direito à nomeação existe quando a Administração Pública viola, por meio de ato expresso ou de subterfúgio, a ordem de classificação prevista no edital ou desconsidera o prazo de validade do certame ainda pendente. Verbete n. 15 da Súmula do STF. Restando comprovado que contratos emergenciais foram realizados e/ou prorrogados dentro do prazo de validade do concurso, está evidenciada a preterição no direito à nomeação di impetrante, violando o artigo 37, incisos IV e IX, da Constituição Federal.(MS. 70022742316, Tribunal Pleno, Tribunal de Justiça do RS., Relator: Arno Werlang, Julgado em 28/04/08(in.www. vlex.com.br/vid/51392291)”

Com essa explanação, só podemos concluir que os nossos Tribunais, vem defendendo nossos cidadãos aprovados regularmente nos concursos públicos e deixando de dar legitimidade a imoralidade administrativa da enxurrada de editais que visam apenas à arrecadação de verba, sem nenhuma responsabilidade no fiel cumprimento do objetivo da administração.

Petruschka Moura Eça da Costa
Advogada inscrita na OAB/RJ, pós-graduada em Direito do Trabalho e Processo do Trabalho pela Universidade Cândido Mendes